Re: [obm-l] Desigualdade

2006-04-20 Por tôpico Fábio Dias Moreira
[EMAIL PROTECTED], 15/04/2006]:

 Sejam p,r,q reais nao-negativos. Tal que p+q+r=1.
 Prove que 7(pq+qr+pr)=2+9pqr.

 [...]

Isso equivale a provar que 7(p+q+r)(pq+qr+rp) = 2(p+q+r)^3 + 9pqr, ou seja,

7(p^2*q + ...) + 21 pqr = 2*(p^3 + q^3 + r^3) + 6(p^2*q + ...) + 21pqr =
2p^3 + 2q^3 + 2r^3 = p^2*q + p*q^2 + q^2*r + q*r^2 + r^2*p + r*p^2

o que é trivial já que p^2*p + q^2*q = p^2*q + q^2*p pela desigualdade do 
rearranjo.

[]s,

-- 
Fábio Dias Moreira
=
Instruções para entrar na lista, sair da lista e usar a lista em
http://www.mat.puc-rio.br/~nicolau/olimp/obm-l.html
=


Re: [obm-l] Divisores de X

2006-01-29 Por tôpico Fábio Dias Moreira
On Mon, 2006-01-30 at 00:41 -0200, Marcelo Salhab Brogliato wrote:
 Se os divisores de um número foram iguais aos divisores de outro
 número, então, eles são o mesmo número.
 [...]

Você resolveu o problema fazendo a suposição (perfeitamente natural) que
D(n) só está definido para n natural (com o perdão do trocadilho). Se n
puder ser negativo, há também a possibilidade x^2 - 1 = 3 - 3x, dando
soluções x = 1 (repetida) ou x = -4 (de fato, D(15) = D(-15)).

[]s,

-- 
Fábio Dias Moreira


=
Instruções para entrar na lista, sair da lista e usar a lista em
http://www.mat.puc-rio.br/~nicolau/olimp/obm-l.html
=


Re: [obm-l] trigo

2005-11-24 Por tôpico Fábio Dias Moreira
On Thu, 2005-11-24 at 11:35 -0200, Rodrigo Augusto wrote:
 bom dia gente,
 
 como eu faço pra calcular o cos de 261º?
 [...]

cos 261 = cos (72 + 72 + 72 + 45). Usando repetidamente a fórmula da
soma, é fácil obter uma expressão explícita para o valor de cos 261, que
dependa apenas de somas, subtrações, multiplicações, divisões e
extrações de raízes quadradas.

Mas é mais fácil comprar uma calculadora científica de dez contos no
camelô mais próximo da sua casa.

[]s,

-- 
Fábio Dias Moreira


=
Instruções para entrar na lista, sair da lista e usar a lista em
http://www.mat.puc-rio.br/~nicolau/olimp/obm-l.html
=


Re: [obm-l] Limite superior para a soma de logaritmos

2005-11-06 Por tôpico Fábio Dias Moreira
On Sun, 2005-11-06 at 00:07 -0200, Denisson wrote:
 Em 06/11/05, Fábio Dias Moreira [EMAIL PROTECTED] escreveu:
  On Sat, 2005-11-05 at 23:37 -0200, Denisson wrote:
   Boa noite pessoal, 
  
   Tou tentando encontrar uma função que limite superiormente a soma
   log1 + log2 + ... + log(n)... e não tou conseguindo...
   Alguma ajuda?
 
  log 1 + log 2 + ... + log n = log n + log n + ... + log n = n * log
  n. 
 
 Tem razão, agora essa função também é limite inferior?

Mais ou menos... Não é muito difícil ver que

log k + log (n-k)  (log n) / 2

para todo n = 3 e 0  k  n. Logo

log 1 + log 2 + ... + log n  (n * log n) / 4

para todo n suficientemente grande.

[]s,

-- 
Fábio Dias Moreira


=
Instruções para entrar na lista, sair da lista e usar a lista em
http://www.mat.puc-rio.br/~nicolau/olimp/obm-l.html
=


Re: [obm-l] Limite superior para a soma de logaritmos

2005-11-06 Por tôpico Fábio Dias Moreira
On Sun, 2005-11-06 at 00:07 -0200, Denisson wrote:
 Em 06/11/05, Fábio Dias Moreira [EMAIL PROTECTED] escreveu:
  On Sat, 2005-11-05 at 23:37 -0200, Denisson wrote:
   Boa noite pessoal, 
  
   Tou tentando encontrar uma função que limite superiormente a soma
   log1 + log2 + ... + log(n)... e não tou conseguindo...
   Alguma ajuda?
 
  log 1 + log 2 + ... + log n = log n + log n + ... + log n = n * log
  n. 
 
 Tem razão, agora essa função também é limite inferior?

Mais ou menos... Não é muito difícil ver que

log k + log (n-k)  (log n) / 2

para todo n = 3 e 0  k  n. Logo

log 1 + log 2 + ... + log n  (n * log n) / 4

para todo n suficientemente grande.

[]s,



 []s,
 
 --
 Fábio Dias Moreira
 
 
 
 =
 Instruções para entrar na lista, sair da lista e usar a lista
 em
 http://www.mat.puc-rio.br/~nicolau/olimp/obm-l.html
 
 =
 
 
 
 -- 
 Denisson
 
 Os homens esqueceram desta verdade; mas tu não a deves esquecer: 
 É só com o coração que se pode ver direito. O essencial é invisível
 aos olhos! (Saint Exupèrry)
-- 
Fábio Dias Moreira


=
Instruções para entrar na lista, sair da lista e usar a lista em
http://www.mat.puc-rio.br/~nicolau/olimp/obm-l.html
=


Re: [obm-l] Limite superior para a soma de logaritmos

2005-11-05 Por tôpico Fábio Dias Moreira
On Sat, 2005-11-05 at 23:37 -0200, Denisson wrote:
 Boa noite pessoal,
 
 Tou tentando encontrar uma função que limite superiormente a soma log1
 + log2 + ... + log(n)... e não tou conseguindo...
 Alguma ajuda?

log 1 + log 2 + ... + log n = log n + log n + ... + log n = n * log n.

[]s,

-- 
Fábio Dias Moreira


=
Instruções para entrar na lista, sair da lista e usar a lista em
http://www.mat.puc-rio.br/~nicolau/olimp/obm-l.html
=


Re: [obm-l] Subconjuntos de R

2005-10-17 Por tôpico Fábio Dias Moreira
[17/10/2005, [EMAIL PROTECTED]:
 O problema a seguir talvez fosse mais desafiador se nao tivesse ainda havido
 esta discussao sobre conjuntos com interior vazio e medida positiva. Apos
 esta discussao, a solucao eh bem obvia:

 Sejam (r_n) uma enumeracao dos racionais, (x_n) uma sequencia de termos
 reais positivos, I_n = (r_n - x_n,  r_n + x_n) e I = Uniao (I_n). Entao, I
 eh um aberto denso em R. Mostre que, se Soma (x_n) convegir, entao I eh um
 subconjunto proprio de R.

 Minha duvida: e se Soma (x_n) divergir? Ainda assim eh possivel termos I
 como subconjunto proprio de R? Neste caso, I = R eh sem duvida possivel.
 Isto
 certamente ocorrerah se tivermos, por exemplo, x_n = r 0 para todo n, sendo
 r constante. Estou analisando esta sitauacao, em que Sona (x_n) diverge.

 Artur
  

Enumere os racionais do intervalo [-3, 3] como p_1, p_2, ... e faça
r_(2^n) = p_n; enumere R \ [-3, 3] com os elementos da enumeração r
que sobraram; escolha x_n = 1/n.

Então o intervalo [-2, 2] só pode ser coberto com intervalos com
centro em [-3, 3]. Mas |I inter [-2, 2]| = 1 + 1/2 + 1/4 + ... = 2 
|[-2, 2]|, logo I != R.

[]s,

-- 
Fábio Dias Moreira


pgpi8418VFsP2.pgp
Description: PGP signature


Re: [obm-l] NOVA trigonometria?

2005-09-17 Por tôpico Fábio Dias Moreira
[17/09/2005, [EMAIL PROTECTED]:
 Um pesquisador (que me pareceu serio) esta propondo uma nova 
 trigonometria supostamente melhor, mais elegante e funcional do que a
 usual. Basicamente ele se propoe e jogar fora os conceitos de seno, 
 cosseno e angulo e distancia (!!)

 Gostaria da opiniao dos participantes da lista. A pagina do cara com
 alguns sample chapters estao em

 http://web.maths.unsw.edu.au/~norman/book.htm
 [...]

A impressão que eu tenho do livro dele é que ele pegou um livro de
geometria e trigonometria e elevou ao quadrado (e eu sinto *muita*
vontade de dizer literalmente).

O livro dele, de um ponto de vista estritamente matemático, parece
correto. Mas eu tenho algumas críticas:

* Nos problemas de trigonometria clássica, ele faz questão de sempre
escrever explicitamente os ângulos envolvidos, mesmo que eles possam
(e em geral, podem) ser expressos como arco senos ou arco cossenos,
afirmando que cálculo é necessário para compreender essas funções.
Isso é patentemente falso -- são raras as ocasiões em que é necessário
calcular explicitamente ângulos, exceto possivelmente para dar a
resposta final -- veja qualquer problema de geometria resolvido por
trigonometria na Eureka! (lembrando que todos esses problemas foram
resolvidos sem calculadora)

* No exemplo que ele dá, que também é criticável segundo os pontos
acima, ele afirma que sqrt(7) é uma grandeza fundamental do problema.
Além de essa ser uma afirmação extremamente vaga (eu poderia afirmar
também que arccos(3/4) é uma grandeza fundamental), o livro erra ao
dizer que ela não aparece na solução tradicional, pois alfa também é
igual a arcsen(sqrt(7)/4).

* Para um livro que começa mencionando tanto a dependência da
geometria tradicional em raízes quadradas, logo o primeiro exemplo dá
uma quadratura que tem uma raiz quadrada (e se o problema fosse um
pouquinho mais complicado, introduziria radicais duplos na história
também).

* E além disso, para calcular o afastamento de duas retas r e s,
sabendo o afastamento de r a t e de s a t, você vai precisar tirar
umas raízes quadradas no meio do caminho, e portanto ter um sinal a
determinar...

Por isso, eu acho que apesar da idéia desse livro ser bonitinha, ela
não tem tantos méritos assim. Mas valeu a tentativa...

[]s,

-- 
Fábio Dias Moreira


pgpH4FtOh48OC.pgp
Description: PGP signature


Re: [obm-l] problema - multiplos de 5, 7, 9 e 11

2005-09-04 Por tôpico Fábio Dias Moreira
[04/09/2005, [EMAIL PROTECTED]:
 olá,

 recebi o problema abaixo de um amigo, e estou tendo dificuldades para
 resolvê-lo.

 seja 'a' um número inteiro positivo tal que 'a' é múltiplo de 5, 'a+1'
 é múltiplo de 7, 'a+2' é múltiplo de 9 e 'a+3' é múltiplo de 11. 
 determine o menor valor que 'a' pode assumir.

Como 5, 7, 9 e 11 são ímpares, é fpacil ver que o problema é
equivalente a achar o menor valor de a tal que 2a é múltiplo de 5, 2a
+ 2 é múltiplo de 7, 2a + 4 é múltiplo de 9 e 2a + 6 é múltiplo de 11.

Nesse problema, obviamente 2a = 5 é a menor solução, logo o menor
valor de 2a possível é 5.

Mas espere aí, você poderia reclamar, isso não dá um valor de a
inteiro!

Essa objeção está perfeitamente correta -- e por isso, precisamos, na
realidade, procurar a menor solução onde 2a é par.

Mas como 5, 7, 9 e 11 são todos primos entre si, a distância entre
soluções consecutivas é 5*7*9*11, logo o próximo valor possível de 2a
é 5 + 5*7*9*11. Logo o menor valor possível de a é (5 + 5*7*9*11)/2 =
1735.

[]s,

-- 
Fábio Dias Moreira


pgpV5ACz7VdUs.pgp
Description: PGP signature


[obm-l] Re: [obm-l] alguém conhece os triangulos e sabe utilizá-los...e não consigui mais material pel o google...

2005-08-27 Por tôpico Fábio Dias Moreira
[27/08/2005, [EMAIL PROTECTED]:
 tipo que matemáticamente vc consegue saber qual sua soma ou então seu
 termo geral...
 ...
 falem alguma aí...só para mim testar pra ver se consigo...

1^1, 2^2, 3^3, 4^4, 5^5, 6^6, 7^7, ..., n^n, ...

[]s,

-- 
Fábio Dias Moreira


pgp8IEtBcmB1Z.pgp
Description: PGP signature


Re: [obm-l] Lógica

2005-08-23 Por tôpico Fábio Dias Moreira

Nicolau C. Saldanha said:
 On Mon, Aug 22, 2005 at 03:57:26PM -0300, Marcelo Roseira wrote:
 [...]

 Sejam as proposições:

 P: Maria está na Itália.
 Q: Jorge é espanhol.
 R: Luíza não está na Itália.

 A proposição composta representada por
 [~(~P#8743;Q)#8743;~R] pode ser descrita por:

 Não entendo estes caracteres estranhos. Suponho que ~ seja NÃO,
 que  seja E mas #8743; eu não sei o que é. Talvez no seu cliente de
 e-mail isto apareça de outra forma?

 [...]


Certamente o #8743; é um e -- isso é a representação do síbolo em HTML.

 [...]

 A) Luíza está na Itália e ou Maria está na Itália ou
 Jorge é espanhol;
 B) Luíza está na Itália e ou Maria está na Itália ou
 Jorge não é espanhol;
 C) Maria e Luíza estão na Itália ou Jorge não é
 espanhol;
 D) Maria e Luíza não estão na Itália e Jorge é
 espanhol;
 E) Maria está na Itália e Luíza não está na Itália e
 Jorge é espanhol.

 Para mim, e ou é um barbarismo, uma afronta tanto à lingua
 portuguesa quanto à lógica.

 [...]

Eu acho (apesar de que eu concordo que é estranho) de que a interpretação
que o enunciado queria era que a alternativa (a) fosse lida como

Luíza está na Itália e (ou Maria está na Itália ou Jorge é espanhol)

onde os parênteses têm o significado matemático (e, no caso, o que está
dentro dos parênteses seria um ou exclusivo).

[]s,

-- 
Fábio Dias Moreira


=
Instruções para entrar na lista, sair da lista e usar a lista em
http://www.mat.puc-rio.br/~nicolau/olimp/obm-l.html
=


Re: [obm-l] Ola podem me ajudar??

2005-08-23 Por tôpico Fábio Dias Moreira

RAfitcho said:
 é muito pratica essa resposta.. mas nao posso escrever isso em uma prova
  discursiva se é que me entende...

 [...]

Porquê não? A solução do Dirichlet está indiscutivelmente certa, e
receberia pontuação máxima em qualquer banca de correção razoável.

(Naturalmente, a questão é *quantas* bancas de correção no Brasil podem
ser consideradas razoáveis...)

[]s,

-- 
Fábio Dias Moreira


=
Instruções para entrar na lista, sair da lista e usar a lista em
http://www.mat.puc-rio.br/~nicolau/olimp/obm-l.html
=


Re: [obm-l] probleminha

2005-08-23 Por tôpico Fábio Dias Moreira

elton francisco ferreira said:
 um agricultor quis repartir as vacas que possuía entre
 seus filhos e notou que se dsse 3 vacas a cada um,
 restariam 24 vacas, e, se lhes desse 7 vacas, daria
 todas as vacas. quantos filhos tinha o agricultor?
 [...]

Imagine que o agricultor distribuiu três vacas para cada filho. Depois de
notar que sobraram 24 vacas, ele pensa:

Poxa! Sobraram tantas vacas! Eu bem que podia ser mais generoso e
distribuir mais vacas para os meus filhos! Ao invés de três, vou dar sete
vacas para cada um!

Ora, se você já tem três vacas, você só precisa de mais quatro vacas para
ficar com sete vacas. Logo, sabemos que o agricultor, depois de dar mais
quatro vacas para cada filho, ficou sem nenhuma sobra -- ou seja,
distribuindo quatro vacas a mais para cada filho, ele distribuiu 24 vacas
no total.

Pergunta: quantos filhos o agricultor tem?

[]s,

-- 
Fábio Dias Moreira


=
Instruções para entrar na lista, sair da lista e usar a lista em
http://www.mat.puc-rio.br/~nicolau/olimp/obm-l.html
=


Re: [obm-l] Problema do casal de filhos

2005-08-21 Por tôpico Fábio Dias Moreira
[21/08/2005, [EMAIL PROTECTED]:
 [21/08/2005, [EMAIL PROTECTED]:
 
  Um homem visita um casal que tem dois filhos. Uma das crianças, um
 menino, vem à sala. Encontre a probabilidade (p) de o outro ser também um
 menino, se
 
  (i) sabe-se que a outra criança é mais nova
 
 (ii) nada se sabe sobre a outra criança
 
  A resposta do item (ii) não é 1/2 Alguém consegue enxergar por
 que
 
 Digamos que H representa um filho homem e M uma filha mulher.
 Como o casal teve dois filhos, as possibilidades são (na ordem mais velha,
 mais nova):

 H, H
 H, M
 M, H
 M, M

 Na primeira situação descrita no problema, sabemos que a criança mais velha
 é um menino. Só podemos ter duas das quatro situações acima:

 H, M
 H, H

 Ou seja, para a outra criança (a mais nova) ser um menino, só há uma
 situação entre duas possíveis. Por isso que a probabilidade é 1/2.

 Na segunda situação, só sabemos que uma das duas crianças é menino. Ou seja,
 das quatro situações possíveis, estamos lidando com apenas três (as que
 possuem no mínimo um H):

 H, H
 H, M
 M, H

 Assim, temos apenas 1 entre 3 possibilidades que satisfazem o enunciado.
 Portanto, para a situação 2, a probabilidade é 1/3.

 []'s
 Kufner
 www.cursinho.hpg.com.br http://www.cursinho.hpg.com.br

Sim, mas nos casos (H, M) e (M, H) a probabilidade do menino, e não a
menina, entrar na sala, é 1/2 (afinal de contas, o enunciado não diz
nada que poderia sugerir uma assimetria entre um eventual menino e uma
eventual menina). Logo os três casos que você mostrou *não têm* a
mesma probabilidade -- a probabilidade desses dois casos é, digamos,
x, e, como a probabilidade de um menino entrar no caso (H, H) é o
dobro da dos outros casos, a probabilidade de (H, H) é 2x. Como a soma
das probabilidades é 1,

x + x + 2*x = 1 =
x = 1/4 =
2*x = 1/2.

Essa é, na realidade, uma aplicação do Teorema de Bayes -- o argumento
que eu fiz acima foi uma versão intuitiva da demonstração formal:

http://mathworld.wolfram.com/BayesTheorem.html

(E, de fato,

(1/4*1)/(1/4*1/2+1/4*1/2+1/4*1) = 1/2

como se poderia esperar.)

[]s,

-- 
Fábio Dias Moreira


pgpFlZRjz0Tts.pgp
Description: PGP signature


[obm-l] Reportagem na TVE

2005-08-01 Por tôpico Fábio Dias Moreira
Oi pessoal,

Assistam, às dez horas da noite de hoje, na TVE, a reportagem da equipe do
Rio que participou da IMC da Bulgária (exceto o Alex, que está de férias
na França).

Assistam também à reportagem do Bujokas, que vai passar também na TVE, às
10h30min da manhã do próximo domingo.

[]s,

-- 
Fábio Dias Moreira


=
Instruções para entrar na lista, sair da lista e usar a lista em
http://www.mat.puc-rio.br/~nicolau/olimp/obm-l.html
=


[obm-l] [OBM-2005] Questão 21/N2, 24/N3

2005-06-14 Por tôpico Fábio Dias Moreira
Oi pessoal,

Eu vi o gabarito da OBM, que dá como resposta correta letra D nesta
questão. O que há de errado com a seguinte solução?

Vamos calcular o número de plins no intervalo (12h, 0h], e descontar os plins
que ocorreram no último segundo depois.

Seja w a velocidade angular do ponteiro das horas. Então as
velocidades angulares dos ponteiros dos minutos e dos segundos são 12w
e 720w.

Vamos contar o número de plins entre cada par de ponteiros:

Minutos/Horas: Do referencial do ponteiro das horas, ele está parado e
o ponteiro dos minutos roda com velocidade angular 11w [*]. Como os
dois começãm juntos, e um ponteiro rodando a w completa uma volta no
período, o ponteiro dos minutos completa 11 voltas nas 12 horas do
problema. Logo há 11 plins gerados por encontros deste tipo.

Segundos/Horas: A velocidade relativa é 720w - w = 719w, logo há 719 plins.

Segundos/Minutos: A velocidade relativa é 720w - 12w = 708w, logo há
708 plins.

Logo, no total, há 11 + 719 + 708 = 1438 plins. Descontando os três
plins ocorridos às 0h, há, no total, 1435 plins no período de 12h1s a
23h59m59s.

[*] Isso não é estritamente verdade do ponto de vista físico, mas não
há importância neste problema.

[]s,

-- 
Fábio Dias Moreira


pgp3gv6ikmhVV.pgp
Description: PGP signature


Re: [obm-l] Desenho do problema (era: Geometria Plana)

2005-05-17 Por tôpico Fábio Dias Moreira
[17/5/2005, [EMAIL PROTECTED]:

 Primeiro nosso heroi faz um desenho bem caprichado do problema,

 Oi, Nicolau:

 É permitido o uso de esquadros, compasso e/ou transferidor na OBM
 ou na IMO (ou em qualquer outra olimpíada)?

Sim; a regra na IMO parece ser um pouco inconsistente -- na IMO do
Japão, o resumo das condições de prova que eles entregaram para a
gente proibia transferidores, mas na IMO da Grécia não parecia haver
esta restrição. Além desse pequeno detalhe, todas as olimpíadas
razoáveis, até onde eu sei, liberam o uso de instrumentos de desenho.

[]s,

-- 
Fábio Dias Moreira


pgpDDXJtH71Q5.pgp
Description: PGP signature


Re: [obm-l] Aritmética

2005-05-04 Por tôpico Fábio Dias Moreira
matduvidas48 said:
 [...]
 04.Um prêmio da sena saiu para dois cartões, um da cidade A e outro da
 cidade B. Nesta última, o cartão era de 6 apostadores, tendo cada um
 contribuído com a mesma importância para a aposta. A fração do prêmio
 total, que cada apostador da cidade B receberá, é: a) 1/6b) 1/8
   c) 1/9  d) 1/10  e) 1/12
 [...]

Cada cartão recebe metade do prêmio, cada apostador da cidade B recebe um
sexto deste dinheiro (pois são seis apostadores). Logo a resposta é
1/2*1/6 = 1/12.

[]s,

-- 
Fábio Dias Moreira


=
Instruções para entrar na lista, sair da lista e usar a lista em
http://www.mat.puc-rio.br/~nicolau/olimp/obm-l.html
=


Re: [obm-l] i^2 = -1 ??

2005-05-01 Por tôpico Fábio Dias Moreira
Bruno Bonagura said:
 Tenho uma breve curiosidade e depois uma pergunta que não achei resposta
 em site nenhum. Primeiro: Quem inventou os números complexos ? Foi o
 Gauss ? Segundo: Quando a teoria dos números complexos foi desenvolvida
 qual foi o axioma base da teoria ? Foi que i² = -1 ? Ou foi imposto que
 a multiplicação de dois complexos implicaria na soma dos ângulos de suas
 representações polares ?
 Essas duas proposições demonstram uma à outra, mas para a teoria imagino
 que uma delas tenha sido adotada como axioma.

 Isso me leva a perguntar oque levou ao desenvolvimento dessa teoria, se
 foi a raiz de números negativos ou se simplesmente foi um conjunto de
 propriedades para facilitar manipulações geométricas no plano dos
 complexos ?

 Um professor meu fazia repetidamente nas aulas o seguinte procedimento:
 (x, y) = (x, 0) + (0, y) = x(1, 0) + y(0, 1) e dizia que o par ordenado
 (1, 0) era despresado/ocutado e o (0,1) definido como i. Então (x, y) =
 x + yi. Realmente é uma explicação que me esclareceu a mente quanto aos
 números complexos. Eles seria apenas vetores que pela imposição da soma
 dos ângulos na multiplicação se chegaria ao i² = -1, ou melhor (0, 1)² =
 (0, -1).

 Queria realmente saber a origem de toda essa teoria.
 [...]

Uma ótima referência sobre o assunto é o livro Meu Professor de
Matemática, do Elon. Em resumo, complexos foram inventados para resolver
equações do terceiro grau -- em alguns casos, é necessário achar as raízes
(e manipulá-las algebricamente) de uma equação do segundo grau com delta 
0 para obter as três raízes (reais!) da equação original.

[]s,

-- 
Fábio Dias Moreira


=
Instruções para entrar na lista, sair da lista e usar a lista em
http://www.mat.puc-rio.br/~nicolau/olimp/obm-l.html
=


RE: [obm-l] Por 7

2005-04-11 Por tôpico Fábio Dias Moreira
Johann Peter Gustav Lejeune Dirichlet said:
 --Que historia e essa de so e bom para numeros
 pequenos? Para comeco de historia, pode-se pegar
 qualquer bloco de digitos em vez de apenas um por vez.
 [...]

Então você certamente acha que 147 não é um múltiplo de 7? Afinal de
contas, 1 - 2*47 = 1 - 94 = 93 = -3*31.

[]s,

-- 
Fábio Dias Moreira


=
Instruções para entrar na lista, sair da lista e usar a lista em
http://www.mat.puc-rio.br/~nicolau/olimp/obm-l.html
=


[obm-l] Re: [obm-l] Média de valores em investimentos

2005-03-24 Por tôpico Fábio Dias Moreira
[24/3/2005, [EMAIL PROTECTED]:
 Senhores, 
 Leia a seguinte suposição:
 Vou fazer um investimento na poupança com os seguintes dados:
 Depósito inicial: R$500,00
 Taxa de Juros: 0.6% a.m.
 Depósitos subsequentes por mês: R$200,00
 Meses de aplicação: 12 meses.
 Resultado: nessas condições terei ao final do perído: R$3018,02 (Hp 12C)

 Observe que esse cálculo é simples de fazer tanto com a
 calculadore HP12C ou na científica, utilizando fórmulas. Mas se os
 depósitos fossem diferentes, por exemplo:
 Mês - Valor
 [...]

 Qual o cálculo que tenho que fazer com os valores dos meses para
 que eu encontre um valor médio de depósitos mensais que corresponda
 ao valores diferentemente depositados? seria alguma média conhecida:
 Aritmética, Geométrica, Harmôncia? ou seja no exemplo acima os
 depósitos dos meses 2 a 12 depois de calculados deverá ter como
 resultado R$200,00 que é o valor fixo que eu depositei no exemplo de
 depósito fixo.

 Alguém se Habilida em responder?

Se você quiser fazer esta conta, você vai precisar de um programa
obscuro, usado apenas pelo marcado financeiro, e que precisa de anos
de treino para ser utilizado corretamente, chamado Microsoft Excel.

[]s,

-- 
Fábio Dias Moreira


pgpvilwvJAy33.pgp
Description: PGP signature


Re: (x+1)^p - x^p - 1 Era:[obm-l] Problemas diversos

2005-03-13 Por tôpico Fábio Dias Moreira
[13/3/2005, [EMAIL PROTECTED]:
 [...]
 Alias, isso me fez pensar nos polinomios F_p(x) = (x+1)^p - x^p - 1, com p 
 primo.
 [...]
 eu cheguei ao seguinte resultado empirico:
 F_p(x) = p*x*(x+1)*(x^2+x+1)^n*G(x), onde G(x) eh um polinomio
 irredutivel sobre Q e n = 1 ou 2, dependendo de p. Mais precisamente:
 n = 1 se p = 5, 11, 17, 23, 29 e 41
 n = 2 se p = 7, 13, 19, 31, 37 e 43
 Perguntas:
 [...]
 2) Os primos para os quais n = 1 sao justamente os primos da forma 6k-1?

Sim -- afinal, (F_p)'(x) = p(x+1)^(p-1) - px^(p-1). Se w = cis 120,
temos que (F_p)'(x) = 0 = (w+1)^(p-1) = w^(p-1). Como p-1 é par,
isso equivale a w^(p-1) = w^(2p-2) = w^(p-1) = 1 = 3 | p-1 =
p == 1 (mod 3). Analogamente, pode-se concluir que (F_p)''(x) != 0 para
todo p.

Como F_p(w) = 0 para todo p, temos que n só pode ser 1, se
p == -1 (mod 3) e 2, se p == 1 (mod 3).

[]s,

-- 
Fábio Dias Moreira


pgptEHdR4uOqQ.pgp
Description: PGP signature


Re: [obm-l] CONJUNTOS (BOAS)

2005-02-26 Por tôpico Fábio Dias Moreira
[26/2/2005, [EMAIL PROTECTED]:
 Me diga uma coisa eu não entendi a do irracional elevado a um irracional...
 
 no caso x = raiz q. de 2 , ae vc eleva os dois lados a raiz q. de 2 , ae fica
 
 x ^raiz q. de 2 = 2, mas isso diz que x ^raiz q. de 2 = 2 é
 racional mas não diz nada a respeito de x, estou certo?

Cuidado -- eu não disse isso; eu *defini* que x = sqrt(2)^sqrt(2).
Por definição, x é um irracional elevado a um irracional. Se x for um
número racional, o problema acabou, pois x é exatamente o que estamos
procurando (um irracional elevado a um irracional dando um racional).
Senão, então x^sqrt(2) é um irracional elevado a um irracional, e
sabemos que este último número vale 2.

Logo, em qualquer uma das hipóteses, conseguimos encontrar um
irracional elevado a um irracional dando um racional. Logo a resposta
ao problema é sim.
 
 Agora me esclareça uma coisa. Veio um arquivo em anexo no
 documento um tal de file... que arquivo é esse??? Meu PC não
 identifica ele acho...

Pode ficar tranqüilo; o arquivo é só a minha assinatura digital.

[]s,

-- 
Fábio Dias Moreira


pgpV5AYWJXPsC.pgp
Description: PGP signature


Re: [obm-l] CONJUNTOS (BOAS)

2005-02-25 Por tôpico Fábio Dias Moreira
[25/2/2005, [EMAIL PROTECTED]:
 6- 39) (ITA-74) Sejam A, B e D subconjuntos não vazios do
 conujunto R dos numeros reais. Sejam as funções f: A - B, (y =
 f(x)), g: D - A (x = g(x)) e a função composta (fog): E - K, Então
 os conjunto E e K são tais que:
 a) E contido A e K contido D
 b) E contido B e K contém A
 c) E contém D, D diferente E e K contido B
 d) E contido D e K contido B 
 e) nenhuma das respostas anteriores
 0bs: assinalei a (d).

Certo.

(Mas eu tenho objeções ao enunciado: em todos os bons livros de
matemática que eu conheço, se f: A - B e g: B - C são funções,
então, *por definição*, a composta de g com f é (gof): A - C.
Naturalmente, podemos restringir o domínio e o contradomínio de gof,
mas isso tem que ser indicado, mesmo que implicitamente.)
 
 *** 8- Um irracional elevado a um irracional pode ser racional?
 (alguem pode mostrar um exemplo e a prova ou só a prova mesmo está
 bom)

Considere x = sqrt(2)^sqrt(2). Se x for racional, acabamos. Senão, x é
irracional. Mas então x^sqrt(2) = [sqrt(2)^sqrt(2)]^sqrt(2) =
sqrt(2)^[sqrt(2)*sqrt(2)] = sqrt(2)^2 = 2, que é claramente racional.

(Para satisfazer a sua curiosidade, x é irracional; na realidade, x é
transcendente -- ou seja, x não é raiz de nenhum polinômio de
coeficientes inteiros.)

[]s,

-- 
Fábio Dias Moreira


pgpjtpwTpkSen.pgp
Description: PGP signature


[obm-l] Re: [obm-l] Re: [obm-l] QUESTÃO DO IME

2005-02-24 Por tôpico Fábio Dias Moreira
[24/2/2005, [EMAIL PROTECTED]:
 sqrt(5 - sqrt(5 - x)) = x.
 sqrt(5 - sqrt(5 - sqrt(5 - sqrt(5 - x = x.
 sqrt(5 - sqrt(5 - sqrt(5 - sqrt(5 - sqrt(5 - sqrt(5 - x)) = x.
 sqrt(5 - sqrt(5 - ... )) = x.
 sqrt(5 - ...) = x.
 sqrt(5 - x) = x.
 5 - x = x^2
 x^2 + x - 5 = 0

 (Resolveu. Equacao do segundo grau.)

 A tecnica consiste em substituir o valor de x infinitamente pela expressão
 que o equivale. Ate vc perceber q todas as parcelas de {sqrt(y)} podem ser
 equivalente a x. Onde y sao as substituicoes infinitas.

Essa solução é realmente muito bonita... É uma pena que esteja errada.

(Se você não acredita, tente resolver a equação

sqrt(0,91 - sqrt(0,91 - x)) = x

com este método.)

[]s,

-- 
Fábio Dias Moreira


pgpmIIdFkbaoF.pgp
Description: PGP signature


Re: [obm-l] Mega Sena

2005-02-24 Por tôpico Fábio Dias Moreira
[24/2/2005, [EMAIL PROTECTED]:
 Eu tava pensando:
 
 Se o cara aposta na mega sena e acerta os 6 numeros ele ganha a
 sena, a quina, a quadra e a trinca? ou somente a Sena?
 Eu acho que é só a Sena. Partindo desse principio:
Se a pessoa na Mega Sena quiser apostar 7 numeros ao inves de
 6, ela paga C(7,6) reais, ou seja, 7 reais. e assim por diante.
 Supondo que ela ganhe o premio maximo apostando 7 numeros em uma
 so cartela, nao seria mais vantajoso ela ter feito as 7 combinacoes
 em cartelas diferentes?? Pois assim ela ganharia nao so na Sena, mas
 na Quina e Quadra também.
 
 [[ Se o processo nao for desse jeito, me desculpem :) ]]

O site da Caixa sugere que não:

http://www.caixa.gov.br/Loterias/Como_Jogar/Asp/Megasena.asp

O que a Caixa parece fazer quando você aposta n dezenas é, na
realidade, ela emite C(n;6) bilhetes com todos os bilhetes de seis
dezenas possíveis. A única razão para se apostar mais de seis dezenas,
portanto, é simplesmente a conveniência que não ter que marcar mais de
um jogo.

(Parece que o valor da Mega-Sena aumentou para R$ 1,50.)

[]s,
 
-- 
Fábio Dias Moreira


pgpXyskgQYnQG.pgp
Description: PGP signature


Re: [obm-l] 3 problemas em aberto

2005-02-22 Por tôpico Fábio Dias Moreira
[22/2/2005, [EMAIL PROTECTED]:
 Restam, na lista, 3 problemas em aberto dentre aqueles propostos na ultima
 semana. O primeiro, que eu propuz, eh de longe o mais facil. [...]

 1) Sao dados n segmentos de reta (cada um de comprimento fixo mas todos
 moveis), os quais, justapostos numa dada ordem, formam um n-gono convexo
 inscritivel.
 Prove que qualquer permutacao desses segmentos formarah um n-gono convexo
 inscritivel e que todos os n-gonos assim formados tem a mesma area (e,
 obviamente, o mesmo perimetro).

Seja R o raio da circunferência circunscrita ao n-ágono, e O o centro
desta circunferência. Se os comprimentos dos lados são l_1, l_2, ...,
l_n e os ângulos associados de vértice O são a_1, a_2, ..., a_n, então
a permutação l_p(1), l_p(2), ..., l_p(n) induz os ângulos a_p(1),
a_p(2), ..., a_p(n). Além disso, como só estamos rearrumando os
triângulos gerados por O e por cada lado, a área é preservada.

  [...] O terceiro dah pra fazer no
 braco, mas obviamente o legal eh achar uma forma esperta de enumerar os
 cortes. Eu pensei no numero de solucoes de x+y+z+w=8 com algumas restricoes
 mas me enrolei.

Se a sua idéia é a que eu estou pensando, o seguinte corte não parece
ser representado por nenhuma solução:


XXOO
XOOX


[]s,

-- 
Fábio Dias Moreira


pgpxTDEAggQXG.pgp
Description: PGP signature


Re: [obm-l] 3 problemas em aberto

2005-02-22 Por tôpico Fábio Dias Moreira
[22/2/2005, [EMAIL PROTECTED]:
 on 22.02.05 13:31, Fábio Dias Moreira at [EMAIL PROTECTED] wrote:
 [22/2/2005, [EMAIL PROTECTED]:
 [...] O terceiro dah pra fazer no
 braco, mas obviamente o legal eh achar uma forma esperta de enumerar os
 cortes. Eu pensei no numero de solucoes de x+y+z+w=8 com algumas restricoes
 mas me enrolei.
 
 Se a sua idéia é a que eu estou pensando, o seguinte corte não parece
 ser representado por nenhuma solução:
 
 
 XXOO
 XOOX
 
 
 []s,

 Precisamente onde eu empaquei. O problema eh aquele X na posicao (2,2) e nao
 adianta girar o quadrado...
 [...]

Acho que eu sei fazer o problema: ao invés de contar cortes, eu vou
contar pinturas do tabuleiro de preto e branco de tal forma que as
duas componentes geradas são conexas e têm a mesma área. Como as
pinturas

 
XXOO OOXX
XOOX OXXO
 

são evidentemente induzidas pelo mesmo corte, temos que dividir o
resultado da contagem por 2 ao final.

A observação inicial é que se as duas componentes são conexas, o corte
só pode tocar a fronteira do tabuleiro duas vezes -- uma para entrar,
outra para sair. Logo, a interseção da componente branca com o anel
formado pelos 12 quadrados exteriores é conexa (e analogamente para a
parte preta):


*..* (anel formado pelos quadrados externos)
*..*


Essa interseção pode ter quatro, cinco, seis, sete ou oito quadrados
brancos, já que as áreas são iguais. Evidentemente, por causa da
dualidade das cores, o número de tabuleiros com quatro quadrados
brancos e com oito quadrados brancos é o mesmo (idem para cinco e
sete).

Caso I -- 4 quadrados brancos:
==

Neste caso, todos os quatro quadrados centrais devem ser brancos, e
basta escolher onde começa a fita de quadrados brancos no anel. Logo
temos 12 possibilidades.

Caso II -- 5 quadrados brancos:
===

Neste caso, três quadrados centrais são brancos, e o formato da fita
externa pode ser de dois tipos, dependendo do ponto de começo desta
(eu estou fixando o sentido horário):

1211
1..2
2..1
1121

# Subcaso 1 -- 8 possibilidades


X..O
X..X


Neste caso, o único caso impossível é o representado no diagrama:


XOXO
XOOX


Logo temos 8*3 = 24 possibilidades neste caso.

# Subcaso 2 -- 4 possibilidades

XOOO
X..O
X..O


Novamente, o único caso impossível é o representado no diagrama:

XOOO
XOXO
XOOO


Logo temos 4*3 = 12 possibilidades neste caso.

No total, temos 24+12 = 36 possibilidades para o caso II.

Caso III -- 6 quadrados brancos:


Neste caso, dois quadrados centrais são brancos, e o formato da fita
externa pode ser novamente de dois tipos:

1121
2..1
1..2
1211

# Subcaso 1: 8 possibilidades


X..O
X..O


Neste caso, as quatro pinturas centrais que não desconectam os
quadrados centrais são possíveis, logo temos 8*4 = 32 possibilidades.

# Subcaso 2: 4 possibilidades


O..O
X..X


A única pintura que não desconecta quadrados que é impossível é esta:


OXXO
XOOX


Logo temos 3*4 = 12 possibilidades.

Logo no caso III temos 32+12 = 44 possibilidades.

==

Logo, no total, temos (12+36+44+36+12)/2 = 140/2 = 70 cortes.

[]s,

-- 
Fábio Dias Moreira


pgpAevOg4IV1l.pgp
Description: PGP signature


Re: [obm-l] novato

2005-02-22 Por tôpico Fábio Dias Moreira
[22/2/2005, [EMAIL PROTECTED]:
 olá  pessoal,  sou  novo  na lista  , entro  aqui com  o intuito 
 de adquirir conhecimento ,  para  pode me  torna uma  pessoal melhor
 , um  abraço  para   todos
 [...]

O Luís Lopes e o Bruno Dias já deram várias referências para
a demonstração da dua desigualdade; se você não recebeu os emails,
veja no arquivo da lista (link no rodapé deste email).

[]s,

-- 
Fábio Dias Moreira


pgp95bnzkPA7o.pgp
Description: PGP signature


Re: [obm-l] Listinha boa!!

2005-02-22 Por tôpico Fábio Dias Moreira
[21/2/2005, [EMAIL PROTECTED]:
 1)As provas de um detonador de granadas efetuam-se no cemtro do
 fundo de um poço cilindrico de profundidade H.Os estilhaços da
 granada, que se produzem depois da explosão e cujas velocidades não
 ultrapassam Vo, não devem cair na superfície da terra.Qual deverá
 ser o diametro minimo d do poço?

Suponha que um estilhaço sai com velocidade inicial que forma um
ângulo w com o plano do fundo do poço. Sejam x_1 e x_2 os
deslocamentos horizontais nos pontos onde o estilhaço está acima da
superfície da terra. Seja ainda r = d/2.

Afirmação: Nenhum estilhaço cai fora do poço se e somente se r  x_1 e
r  x_2 para todos os possíveis x.

Demonstração: A volta da propsição é bem óbvia -- neste caso, os
estilhaços voam acima da superfície mas não tem potência suficiente
para cair fora do poço. Além disso, se x_1  r  x_2, o estilhaço
efetivamente cai na superfície. Resta estudar o caso onde r  x_1 e r
 x_2, que é o caso onde o estilhaço bate na parede do poço. Mas neste
caso, aumentar o valor de w aumenta a altura máxima e reduz o alcance
do estilhaço. Como, para w = pi/2, x_1 = x_2 = 0, pelo TVM, existe w
tal que x_1  r  x_2, logo algum estilhaço cai fora do poço.

Fixando o referencial no fundo do poço, temos que a equação do
movimento é

x(t) = t * v_0 * cos w
y(t) = t * v_0 * sen w - g * t^2 / 2.

Então x_1 e x_2, se existirem, são as raízes de y(t) = H.
Convencionando x_1  x_2, é fácil ver que

x(t_2) = [v_0 * cos w / g]*[v_0 * sen w + sqrt(v_0^2 * sen^2 w - 2 * g
* H)].

Chamando k^2 de 2*g*H/v_0^2, o nosso problema se reduz a achar o máximo
de

cos w*[sen w + sqrt(sen^2 w - k^2)]. Esse máximo será o valor de r.
Como 0  w  pi/2, os extremos não maximizam a função e cos w = sqrt(1
- sen^2 w). Chamando sen w de u (logo 0  u  1), temos que maximizar

sqrt(1 - u^2)*(u + sqrt(u^2 - k^2)).

Apesar que eu não fiz a conta, não parece ser muito fácil achar esse
máximo -- igualar a derivada a zero na mão é impraticável.

[]s,

-- 
Fábio Dias Moreira


pgpzdkVq7aXv4.pgp
Description: PGP signature


Re: [obm-l] ajudinha básica com complexos

2005-02-17 Por tôpico Fábio Dias Moreira
Thiago Addvico escreveu:
é algo bem simples, mas eu estou me atrapalhando muito nas soluções,
achando coisas q divergem dos resultados do livro:
Determine Z pertencente ao conjunto dos complexos tal que z elevado ao
cubo é igual ao conjugado de Z
[...]
Seja z* o conjugado de z. Então temos z^3 = z*. Aplicando o módulo
dos dois lados, |z|^3 = |z| = |z| = 0 ou |z| = 1. O caso |z| = 0
implica z = 0. O caso |z| = 1 implica z^3 = z* = z^4 = z*z = |z|^2
= 1 = z^4 = 1. Como todas as passagens são equivalências, as
soluções são 0, 1, -1, i, -i.
[...]
Sendo x^2 + y^2 = 1, Prove que (1 + x + y . i)/(1 + x - y . i) = x + y . i
[...]
Isso não faz sentido no caso x = -1 e y = 0.
[]s,
--
Fábio Dias Moreira
=
Instruções para entrar na lista, sair da lista e usar a lista em
http://www.mat.puc-rio.br/~nicolau/olimp/obm-l.html
=


Re: [obm-l] Moedas em sacos

2005-02-16 Por tôpico Fábio Dias Moreira
Fábio Dias Moreira escreveu:
Rogerio Ponce escreveu:
Ola' Qwert, Bruno, Claudio e colegas da lista,
o fato e' que N pode ser ainda maior que 927...
[...]

Considere todos os ternos (p, q, r) de inteiros com |p|, |q|, |r| =
10 e tais que mdc(p, q, r) = n (estou definindo mcd(x, 0) = |x|).
Seja S o conjunto desses ternos. Eu afirmo que é possível fazer o
pedido com N = #S.
[...]
Logo
N = #S_1 = 9261 - 1331 - 343 - 125 - 27 + 27 + 27 + 1 = 7490.
Isso também prova que, se todas as pesagens forem balanceadas, essa
*é* a cota superior, logo basta provar que pesagens não-balanceadas
não permitem ir além de um limite inferior a 7490.
[...]
Desculpem -- eu quero dizer 7491. O terno (0, 0, 0) pode ser 
adicionado a S sem risco de ser confundido com algum dos outros ternos.

[]s,
--
Fábio Dias Moreira
=
Instruções para entrar na lista, sair da lista e usar a lista em
http://www.mat.puc-rio.br/~nicolau/olimp/obm-l.html
=


Re: [obm-l] Problema

2005-02-16 Por tôpico Fábio Dias Moreira
Bruno Bruno escreveu:
Olha, esse enunciado não é verdade. Podemos dipor as moedas da seguinte maneira:
 0
0 0
   0   0
  0 0
 0   0
0 0 0 0 0 0
Essa disposição é perfeitamente viável e não contraria o enuunciado
(15 moedas de mesmo diametro formando um triangulo equilatero).
Ora, usando essa convençao de X e O, podemos botar as moedas assim:
 O
X X
   O   X
  X O
 O   X
X X O X O O
Não formando assim, qualquer triangulo equilatero (não estou querendo
ser pentelho, mas é que realmente eu pensei nessa configuraçao antes
de pensar naquela onde estao todas amontoadas).
[...]
Bom, eu também pensei nessa distribuição inicialmente, e até achei
um contraexemplo -- por isso que eu fiz o problema com a
distribuição que eu mostrei. Mesmo assim, o seu exemplo contém
vários triângulos equiláteros monocromáticos (onde?).
[]s,
--
Fábio Dias Moreira
=
Instruções para entrar na lista, sair da lista e usar a lista em
http://www.mat.puc-rio.br/~nicolau/olimp/obm-l.html
=


Re: [obm-l] Teo de fermat provado com matematica elementar?

2005-02-15 Por tôpico Fábio Dias Moreira
-BEGIN PGP SIGNED MESSAGE-
Hash: SHA1
Fabio Niski escreveu:
| Por gentileza senhores, alguem poderia comentar sobre esta suposta
prova
| usando apenas conceitos do ensino medio?
|
| http://xxx.lanl.gov/abs/math.GM/0502245
| [...]
Eu acho que a afirmação imediatamente após a equação 35 é um erro
fatal, já que n não precisa ser par.
(Apesar de que não há nenhum problema imediatamente, ele explora o
fato de que a raiz (n/2)-ésima de (1/X + 2X + 2) é inteiro somente
se (1/X + 2X + 2) é inteiro, o que se torna falso no caso n ímpar.)
[]s,
- --
Fábio Dias Moreira
-BEGIN PGP SIGNATURE-
Version: GnuPG v1.4.0 (MingW32)
Comment: Using GnuPG with Thunderbird - http://enigmail.mozdev.org
iD8DBQFCEmmGp7qMXa2oQtsRAtKyAJ0eDAThajOsD8c4IEixJY6+9dAx4ACfex9s
Og/Sxf5reOWucZCHehNpTSM=
=O0ri
-END PGP SIGNATURE-
=
Instruções para entrar na lista, sair da lista e usar a lista em
http://www.mat.puc-rio.br/~nicolau/olimp/obm-l.html
=


Re: [obm-l] Problema

2005-02-15 Por tôpico Fábio Dias Moreira
benedito escreveu:
Quinze moedas de mesmo diâmetro são dispostas formando um triângulo 
eqüilátero. As faces de cada uma das moedas são pintadas ou de branco ou 
de preto. Prove que, qualquer que seja a pintura, existem três moedas de 
mesma cor cujos centros são vértices de um triângulo eqüilátero.
[...]
Suponha que não há triângulo equilátero e considere o tabuleiro:
.
   . .
  . a .
 . a a .
. . . . .
Os três quadrados marcados com a não podem ser da mesma cor. Suponha
s.p.d.g. que eles são pintados da seguinte forma:
.
   . .
  . O .
 . X X .
. . 1 . .
(Eu estou seguindo a convenção do Go -- O é branco, X é preto)
O ponto 1 deve ser branco:
.
   . .
  . O .
 2 X X 2
. . O . .
Os pontos 2 têm que ser pretos:
.
   . .
  . O .
 X X X X
. 3 O 3 .
Os pontos 3 têm que ser brancos:
.
   . .
  . * .
 X X X X
. * O * .
Mas então acabamos de formar um triângulo equilátero nos três pontos
marcados.
[]s,
--
Fábio Dias Moreira
=
Instruções para entrar na lista, sair da lista e usar a lista em
http://www.mat.puc-rio.br/~nicolau/olimp/obm-l.html
=


Re: [obm-l] Moedas em sacos

2005-02-15 Por tôpico Fábio Dias Moreira
Rogerio Ponce escreveu:
Ola' Qwert, Bruno, Claudio e colegas da lista,
o fato e' que N pode ser ainda maior que 927...
[...]
Considere todos os ternos (p, q, r) de inteiros com |p|, |q|, |r| =
10 e tais que mdc(p, q, r) = n (estou definindo mcd(x, 0) = |x|).
Seja S o conjunto desses ternos. Eu afirmo que é possível fazer o
pedido com N = #S.
Para ver isso, faça uma bijeção entre os sacos e os ternos de S. Na
i-ésima pesagem, coloque t_i moedas do saco associado a t no prato
da direita (faça a coisa natural no caso t_i  0). Como t pertence a
S = -t pertence a S, a balança acusa um valor de t_i*d, onde d é a
diferença de peso entre as moedas defeituosas. Logo as três pesagens
revelarão o valor de t*d. Como d  0 e as três componentes de t são
primas entre si, o mdc real entre as três componentes de t é
exatamente d, logo é possível achar t.
Agora o problema é achar N. Pelo PIE, não é difícil ver que
#S = 21^3 - #T_2 - #T_3 - #T_5 - #T_7 + #T_6 + #T_10 + 1
onde #T_n é o conjunto dos ternos com norma do sup = 10 e o mcd
entre as componentes é um múltiplo de n.
Então
#T_2 = 11^3
#T_3 = 7^3
#T_5 = 5^3
#T_6 = #T_7 = #T_10 = 3^3
Logo
N = #S_1 = 9261 - 1331 - 343 - 125 - 27 + 27 + 27 + 1 = 7490.
Isso também prova que, se todas as pesagens forem balanceadas, essa
*é* a cota superior, logo basta provar que pesagens não-balanceadas
não permitem ir além de um limite inferior a 7490.
(O meu raciocínio está certo? A contagem está certa; eu conferi com 
um programinha em Python.)

[]s,
--
Fábio Dias Moreira
=
Instruções para entrar na lista, sair da lista e usar a lista em
http://www.mat.puc-rio.br/~nicolau/olimp/obm-l.html
=


Re: [obm-l] Senos e cossenos estranhos...

2005-02-13 Por tôpico Fábio Dias Moreira
-BEGIN PGP SIGNED MESSAGE-
Hash: SHA1
carlos gomes escreveu:
| Algum colega pode me ajudar com essa:
|
| Suponha que x, y, z e w são números reais tais que:
|
| senx+seny+senz+senw=0
| cosx+cosy+cosz+cosw=0
|
| Mostre que :
|
| (senx)^2003+(seny)^2003+(senz)^2003+(senw)^2003=0
| [...]
Definição: S_r = {v | |v| = r} e B_r = {v | |v| = r} (ou seja, S_r e
B_r são a circunferência e a bola fechada de raio r.
Lema: Todos os vetores de B_2\{0} se escrevem unicamente (a menos da
ordem dos elementos) como soma de dois elementos de S_1.
Demonstração: Pela lei dos cossenos, se u e v estão em S_1 e fazem um
ângulo de x, então |u+v| = sqrt(2 + 2cos x), que é uma bijeção do
intervalo [0, pi) no intervalo (0, 2]. Aplicando uma rotação conveniente
a u e v, podemos fazer u e v assumir qualquer vetor de S_{sqrt(2 + 2cos
x)}. Logo o lema está demonstrado.
Considere 4 vetores a, b, c e d em S_1, tais que a+b+c+d = 0. (note que
os argumentos desses vetores satisfazem as hipóteses do enunciado, já
que as coordenadas dos vetores podem ser expressas como (sen x, cos x) e
assim sucessivamente).
Então, a+b = -(c+d). Mas (-c) + (-d) = -(c+d) = a + b, logo, pelo lema,
podemos supor s.p.d.g. que a = -c e, analogamente, b = -d. Logo, se x,
y, z, w são os argumentos de a, b, c e d, temos que sen x = -sen z, cos
x = -cos z, sen y = -sen w e cos y = -cos w, e segue trivialmente que
(sen x)^2003 + (sen y)^2003 + (sen z)^2003 + (sen w)^2003 =
(sen x)^2003 + (sen y)^2003 - (sen x)^2003 - (sen y)^2003 = 0.
(A demonstração acima tem um pequeno erro que não afeta a
afirmação-chave do problema, mas que precisa ser corrigido. Qual?)
[]s,
- --
Fábio Dias Moreira
-BEGIN PGP SIGNATURE-
Version: GnuPG v1.4.0 (MingW32)
Comment: Using GnuPG with Thunderbird - http://enigmail.mozdev.org
iD8DBQFCD15kp7qMXa2oQtsRAodZAJ9owFdE+jQYG+UzWIIDuPfdRdi8fACfWTHz
MFL9FSDpOxZJkJU8qTEAyig=
=L5CT
-END PGP SIGNATURE-
=
Instruções para entrar na lista, sair da lista e usar a lista em
http://www.mat.puc-rio.br/~nicolau/olimp/obm-l.html
=


Re: [obm-l] Eureka 02: No mínimo 21 números

2005-02-11 Por tôpico Fábio Dias Moreira
-BEGIN PGP SIGNED MESSAGE-
Hash: SHA1
[EMAIL PROTECTED] escreveu:
| Olá pessoal !
|
| Escolha um número de quatro dígitos (nenhum deles zero) e começando com
| ele construa uma lista de 21 números distintos, de quatro dígitos cada
| um, que satisfaça a seguinte regra: depois de escrever cada novo número
| da lista devem-se calcular todas as médias entre dois dígitos desse
| número, descartando-se as médias que não dão um número inteiro, e com os
| que restam se forma um número de quatro dígitos que ocupará o lugar
| seguinte na lista. Por exemplo, se na lista se escreveu o número 2946, o
| seguinte pode ser  ou 3434 ou 5345 ou qualquer outro número armado
| com os dígitos 3, 4 ou 5.
| [...]
Você já sabe construir uma lista com 6 números? 12 números? 18 números?
Não é muito difícil ver que a lista sempre acaba (se você for esperto)
em um número de quatro dígitos iguais. Portanto, você quer começar com
um número que tenha a maior energia potencial possível.
(Eu me lembro de ter feito essa questão da prova -- sim, eu estou
ficando velho -- e eu tenho a impressão de que esse 21 pode ser refinado
para 29. Eu estou falando besteira?)
[]s,
- --
Fábio Dias Moreira
-BEGIN PGP SIGNATURE-
Version: GnuPG v1.4.0 (MingW32)
Comment: Using GnuPG with Thunderbird - http://enigmail.mozdev.org
iD8DBQFCDYwUp7qMXa2oQtsRAsNcAJ9syKfaDU0fPE+l22hjzmwLnkB6xwCdEd7A
jQ/8WqObxbp3THbaYbwJuNM=
=suNw
-END PGP SIGNATURE-
=
Instruções para entrar na lista, sair da lista e usar a lista em
http://www.mat.puc-rio.br/~nicolau/olimp/obm-l.html
=


Re: [obm-l] Exercícios...

2005-02-05 Por tôpico Fábio Dias Moreira
Vinícius Meireles Aleixo wrote:
Oi, boa noite
Aqui vão alguns exercicios do meu livro de cálculo(derivadas) que não consegui 
fazer, caso possam da uma ajudinha...
[...]

[...]
2-Prove que a mais curta distancia de um ponto (X_1, Y_1) ao gráfico de uma função 
diferenciável f é medida ao longo de uma normal ao gráfico, isto é, uma 
perpendicular à tangente.
[...]
Isso é falso: Tome f: (0, 1) - (0, 1), f(x) = x e (x_1, y_1) = (2, 2).
[]s,
--
Fábio Dias Moreira
=
Instruções para entrar na lista, sair da lista e usar a lista em
http://www.mat.puc-rio.br/~nicolau/olimp/obm-l.html
=


Re: [obm-l] SISTEMA

2004-09-29 Por tôpico Fábio Dias Moreira
-BEGIN PGP SIGNED MESSAGE-
Hash: SHA1

samanta [EMAIL PROTECTED] said:
 Olá Fabio,

 Infelizmente, eu não consegui entender o início:

 Fatorando a segunda equação, b(3a^2 - b^2) = -2. Se a e b forem inteiros,
 há quatro possibilidades para o b, que determinam o valor de a. Delas,
 apenas b = 2 e b = -1

 Como você encontrou esses valores?

 Obrigada pela atenção,
 []´s Samanta
 [...]

Só existem quatro divisores inteiros de -2: -2, -1, 1 e 2, logo b deve ser um 
deles.

b = -2: -2(3a^2 - 4) = -2 = 3a^2 = 5, impossível
b = -1: -1(3a^2 - 1) = -2 = 3a^2 = 3 = a^2 = 1.
b =  1:  1(3a^2 - 1) = -2 = 3a^2 = -3, impossível.
b =  2:  2(3a^2 - 4) = -2 = 3a^2 = 3 = a^2 = 1.

[]s,

- -- 
Fábio Dias Moreira
-BEGIN PGP SIGNATURE-
Version: GnuPG v1.2.3 (GNU/Linux)

iD8DBQFBWolZalOQFrvzGQoRAgHEAJ0X3oHHN76mI1WRyN04usETNBympwCeMv2b
P4GZD/sL8+ZMRUicv4G8ZJ8=
=b8Mu
-END PGP SIGNATURE-


=
Instruções para entrar na lista, sair da lista e usar a lista em
http://www.mat.puc-rio.br/~nicolau/olimp/obm-l.html
=


Re: [obm-l] SISTEMA

2004-09-27 Por tôpico Fábio Dias Moreira
-BEGIN PGP SIGNED MESSAGE-
Hash: SHA1

samanta [EMAIL PROTECTED] said:
 Olá amigos,
 Existe solução para esse sistema?

 a^3 - 3a(b^2) = -11
 3(a^2) - b^3 = -2
 [...]

O Leandro já respondeu à sua pergunta, mas eu acho que você queria dizer

a^3 - 3ab^2 = -11
3a^2b - b^3 = -2

i.e. você quer achar a raiz cúbica de -11-2i.

Fatorando a segunda equação, b(3a^2 - b^2) = -2. Se a e b forem inteiros, há 
quatro possibilidades para o b, que determinam o valor de a. Delas, apenas b 
= 2 e b = -1 geram a inteiro (e nos dois casos, a^2 = 1).

Substituindo na equação de cima, a(1 - 3b^2) = -11. A única possibilidade é b 
= 2 e a = 1, logo (1 + 2i)^3 = -11-2i. As outras duas soluções do sistema são 
geradas multiplicando por cis 120.

[]s,

- -- 
Fábio Dias Moreira
-BEGIN PGP SIGNATURE-
Version: GnuPG v1.2.3 (GNU/Linux)

iD8DBQFBWIxBalOQFrvzGQoRAu6kAKC4zFLL/ZkyBKtd3eScxKwuH7e1PgCeKxxc
bwclvxj+8oLHqhyJD1V3knI=
=ze3k
-END PGP SIGNATURE-


=
Instruções para entrar na lista, sair da lista e usar a lista em
http://www.mat.puc-rio.br/~nicolau/olimp/obm-l.html
=


Re: [obm-l] PESQUISA ELEITORAL!

2004-08-19 Por tôpico Fábio Dias Moreira
-BEGIN PGP SIGNED MESSAGE-
Hash: SHA1

[EMAIL PROTECTED] said:
 [...]
 A propósito, o que é absurdo nesta declaração Metade dos entrevistados,
 tiveram um desempenho abaixo da média
 [...]

Além da vírgula separando sujeito e predicado? Nada; se a distribuição for 
simétrica, a afirmação é perfeitamente válida.

[]s,

- -- 
Fábio Dias Moreira
-BEGIN PGP SIGNATURE-
Version: GnuPG v1.2.3 (GNU/Linux)

iD8DBQFBJUYhalOQFrvzGQoRAi8IAKCVSfRgKsqD/Lapq3rx6WCsG8a86ACdHP9B
5O/puWUpP4Hfh9vU+1Uz6xk=
=rRe0
-END PGP SIGNATURE-


=
Instruções para entrar na lista, sair da lista e usar a lista em
http://www.mat.puc-rio.br/~nicolau/olimp/obm-l.html
=


Re: [obm-l] Prova da IMC - 1o. dia (correcao)

2004-07-25 Por tôpico Fábio Dias Moreira
-BEGIN PGP SIGNED MESSAGE-
Hash: SHA1

[EMAIL PROTECTED] said:
 [...]
 3) Let S_n be the set of all sum x_1+x_2+...x_n, where
 n=2, 0=x_1,...,x_n=pi/2 and
 sin(x_1) + sin(x_2) + ... + sin(x_n) = 1
 a) Show that S_n is an interval.
 b)Let l_n be the length of S_n. Find lim(n-infinito)(l_n).
 [...]

[espaço para quem quer pensar no problema]



















































a) Seja y_i = sen(x_i). Então x_1 + ... + x_n = arcsen(y_1) + ... + 
arcsen(y_n), já que os x_i estão restritos ao intervalo [0, pi/2].

Tome então a função f: S - R que leva (y_1, ..., y_n) em arcsen(y_1) + ... + 
arcsen(y_n), e S é o conjunto dos vetores (y_1, ..., y_n) do R^n tais que:

* y_1 + ... + y_n = 1
* 0 = y_i = 1 para todo i, 1 = i = n.

f é obviamente contínua, e S é obviamente conexo, logo f(S) é conexo, logo é 
um intervalo.

b) Lema: f(S) = [n*arcsen(1/n), pi/2]

Prova: O resultado é obviamente verdadeiro para n = 1. Suponha que ele é 
válido para n-1.

Pelo teorema do Multiplicador de Lagrange, o único ponto crítico da f é o 
ponto (1/n, 1/n, ..., 1/n) (pois df/dy_i = 1/sqrt(1-y_i^2), e os pontos 
críticos da f são caracterizados por grad(F) // (1, 1, ..., 1), pois o vetor 
(1, 1, ..., 1) é o vetor normal à superfície S). Portanto, um candidato a 
ponto de mínimo é (1/n, ..., 1/n) (com valor n*arcsen(1/n)), já que é fácil 
ver que a Hessiana de f é uma matriz com zeros fora da diagonal e entradas da 
forma y_i/(1-y_i^2)^(3/2) na diagonal, logo é uma matriz positiva definida.

Por outro lado, existem dois tipos de pontos na fronteira de S:

* pontos com algum y_i = 1;
* pontos com algum y_i = 0.

Se y_i = 1, então y_1 = y_2 = ... = y_{i-1} = y_{i+1} = ... = y_n = 0, e a 
função vale pi/2. Mas se y_i = 0, a função f se comporta exatamente como no 
caso n-1. Logo, pela hipótese de indução, a imagem da fronteira de S por f é 
[(n-1)*arcsen(1/(n-1)), pi/2]. Logo o máximo da f, em dimensão n, é pi/2, e o 
seu mínimo é n*arcsen(1/n), pois a derivada de x*arcsen(1/x), arcsen(1/x) - 
1/sqrt(x^2-1), é sempre negativa se x  1, pois

arcsen(1/x)  1/sqrt(x^2-1) =
1/x  sen(1/sqrt(x^2-1)), verdadeiro pois sen(1/sqrt(x^2-1))  1/sqrt(x^2-1)  
1/sqrt(x^2) = 1/x. Em particular, n*arcsen(1/n)  (n-1)*arcsen(1/(n-1)).

Logo l_n = pi/2 - n*arcsen(1/n). Mas arcsen(1/n) = 1/n + O(1/n^2), logo
lim[n-inf] l_n = lim[n-inf] pi/2 - 1 + O(1/n) = pi/2 - 1.

[]s,

- -- 
Fábio Dias Moreira
-BEGIN PGP SIGNATURE-
Version: GnuPG v1.2.3 (GNU/Linux)

iD8DBQFBA/QoalOQFrvzGQoRAuTVAJ45AUYJiaCRJlrEKT/5eW9bEUdphQCdHAY5
Aaz53/HZ7CtSOOaV5eatX7k=
=sbcC
-END PGP SIGNATURE-


=
Instruções para entrar na lista, sair da lista e usar a lista em
http://www.mat.puc-rio.br/~nicolau/olimp/obm-l.html
=


Re: [obm-l] Série Infinita

2004-07-23 Por tôpico Fábio Dias Moreira
-BEGIN PGP SIGNED MESSAGE-
Hash: SHA1

Flávio Ávila [EMAIL PROTECTED] said:
 Submeto o seguinte problema:

 Calcule o limite da seqüência, quando n tende a infinito:  Sn = 1/n +
 1/(n+1) + ... + 1(2*n).
 
 Eu acho que já consegui resolver este problema, mas foi há muito e tempo
 atrás, e não me lembro como o fiz.  Se não me engano o resultado é ln(2).
 [...]

Como, essencialmente, 1/1 + 1/2 + ... + 1/n = ln n + c + O(1/n), onde c é uma 
constante positiva, temos que S_n = ln 2n + c + O(1/n) - ln n-1 - c - O(1/n) 
= ln[2n/(n-1)] + O(1/n), que obviamente tende a ln 2.

[]s,

- -- 
Fábio Dias Moreira
-BEGIN PGP SIGNATURE-
Version: GnuPG v1.2.3 (GNU/Linux)

iD8DBQFBAd93alOQFrvzGQoRAkeYAJ9v2UrI4YLsAiwqU+EJTFwjpEsPAQCgyPnQ
PGDJgOwNuvZP2haL4NOOgLM=
=Mu7n
-END PGP SIGNATURE-


=
Instruções para entrar na lista, sair da lista e usar a lista em
http://www.mat.puc-rio.br/~nicolau/olimp/obm-l.html
=


Re: [obm-l] RES: [obm-l] calculo de área - acho que precisa de integral

2004-07-21 Por tôpico Fábio Dias Moreira
-BEGIN PGP SIGNED MESSAGE-
Hash: SHA1

Ralph Teixeira [EMAIL PROTECTED] said:
   Se eu entendi direito o problema... Quadrado ABCD de lado a, centro O,
 círculo de centro A e raio a, círculo de centro O e raio a/2. Sejam E e F
 os pontos de interseção das duas circunferências. Então o problema é
 encontrar  área da lua entre os dois arcos EF, é isto?
 [...]
   Total: a^2/8*(sqrt(7)+2pi-2x-8y) onde x e y são calculáveis como acima
 (arccos disso e daquilo). Confira aí se eu errei alguma conta -- o método
 certamente funciona, mas é difícil saber se a resposta poderia ser mais
 simplificada [...]

A gente já resolveu esse problema no treinamento de segunda-feira no IMPA, e a 
resposta é um número feio que nem esse aí mesmo.

[...] (eu tentei calcular cos(8y+2x) para ver se 8y+2x era um ângulo
 conhecido, mas deu 393/4096, que não me parece ser o cosseno de um ângulo
 conhecido). :P
 [...]

E não pode ser mesmo -- um dos problemas propostos da Eureka! 17 é provar que 
cos(m*pi/n) é racional somente se |n| = 3, logo os ângulos de 60 e 90 graus 
são, essencialmente, os únicos que têm cosseno racional.

[]s,

- -- 
Fábio Dias Moreira
-BEGIN PGP SIGNATURE-
Version: GnuPG v1.2.3 (GNU/Linux)

iD8DBQFA/wV2alOQFrvzGQoRArxeAKDQz8MJD43ToTACRvIlojhozbHfdwCgpj6F
NM4cO+IqsxCYbA1hdyeka/c=
=KOVl
-END PGP SIGNATURE-


=
Instruções para entrar na lista, sair da lista e usar a lista em
http://www.mat.puc-rio.br/~nicolau/olimp/obm-l.html
=


Re: [obm-l] Função Exponencial

2004-07-20 Por tôpico Fábio Dias Moreira
-BEGIN PGP SIGNED MESSAGE-
Hash: SHA1

Lista OBM [EMAIL PROTECTED] said:
 Gostaria de saber se existe duas funções reais f e g tais que (fog)(x) =
 e^x.
 [...]

Como outros já responderam, sim, existe: basta tomar f(x) = x e g(x) = e^x.

O mais interessante nesse problema é que existe uma função f: R - R tal que 
(fof)(x) = e^x -- esse é um dos problemas propostos na Matemática 
Universitária, no. 35, páginas 41-46.

(espaço para quem quer pensar no problema...)


















































Cosndiere A_1 = (-1, 0], A_2 = (-inf, -1] e, se A_i = (a_i, b_i], então 
A_{i+2} = (e^a_i, e^b_i] (estou definindo e^-inf = 0). É fácil ver que os 
A_i's são uma partição de R.

Agora, defina f_i: A_i - A_{i+1} por f_1(x) = -1/(x+1) e f_{i+1}(x) = 
e^(f_i^{-1}(x)), onde f_i^{-1} é a inversa da f_i. Para provar que esta 
definição faz sentido, temos que provar que f_i é invertível para todo i. 
Isso é verdade para i = 1; suponha a afirmação verdadeira para f_{i-1}. Então 
f_i é trivialmente injetora, e é sobrejetora, pois a imagem de f_{i-1}^{-1} é 
A_{i-1}, logo a imagem de f_i é a exponencial de A_{i-1}, que é A_{i+1}.

Finalmente, defina f(x) = f_i(x), onde i é escolhido de tal forma que x 
pertença a A_i. Então

f(f(x)) = f(f_i(x)). Mas f_i(x) pertence a A_{i+1}, logo

f(f(x)) = f_{i+1}(f_i(x)) = e^(f_i^{-1}(f_i(x))) = e^x para todo x real.

[]s,

- -- 
Fábio Dias Moreira
-BEGIN PGP SIGNATURE-
Version: GnuPG v1.2.3 (GNU/Linux)

iD8DBQFA/aOmalOQFrvzGQoRApaJAJwOwqqzb2/iF37X4BnJ+fPFyHZylQCePqdA
Z9SahgcKCY+ovHQkGILqRWg=
=EbqB
-END PGP SIGNATURE-


=
Instruções para entrar na lista, sair da lista e usar a lista em
http://www.mat.puc-rio.br/~nicolau/olimp/obm-l.html
=


Re: [obm-l] problema combinatoria

2004-06-23 Por tôpico Fábio Dias Moreira
-BEGIN PGP SIGNED MESSAGE-
Hash: SHA1

rafaelc\.l [EMAIL PROTECTED] said:
  De quantos modos 720 pode ser dividido em um produto
 de três inteiros positivos?
 [...]

Como 720 = 2^4*3^2*5, a*b*c = 720 ==

a = 2^a1*3^a2*5^a3
b = 2^b1*3^b2*5^b3
c = 2^c1*3^c2*5^c3

com a1+b1+c1 = 4, a2+b2+c2 = 2, a3+b3+c3 = 1. Essas três equações são 
independentes e têm C(6;2), C(4;2), C(3;2) soluções, respectivamente. Logo o 
número total de maneiras é 6*5*4*3*3*2/2^3 = 270.

[]s,

- -- 
Fábio Dias Moreira
-BEGIN PGP SIGNATURE-
Version: GnuPG v1.2.3 (GNU/Linux)

iD8DBQFA2aagalOQFrvzGQoRAoZPAJ4tTORpddz1suZ5PgwjNI3fXwoDZgCfd1ai
MDotoc4wqsvrRLgfbcZoiI8=
=CCZ9
-END PGP SIGNATURE-


=
Instruções para entrar na lista, sair da lista e usar a lista em
http://www.mat.puc-rio.br/~nicolau/olimp/obm-l.html
=


Re: [obm-l] poligono

2004-06-20 Por tôpico Fábio Dias Moreira
-BEGIN PGP SIGNED MESSAGE-
Hash: SHA1

Augusto Cesar de Oliveira Morgado [EMAIL PROTECTED] said:
 (F) Todas sao falsas.

 22,5 graus eh o angulo central de um poligono de 16 lados. Se X e Y sao
 vertices consecutivos, n=16. Se nao forem consecutivos, n serah multiplo de
 16. [...]

Mas o ângulo inscrito na circunferência vale metade do ângulo central, logo 
basta que 8|n, não? O ponto O é um dos vértices do polígono, não o seu 
centro.

[]s,

- -- 
Fábio Dias Moreira
-BEGIN PGP SIGNATURE-
Version: GnuPG v1.2.3 (GNU/Linux)

iD8DBQFA1hbEalOQFrvzGQoRAv+CAKDKLXKCnP5VutDVaxfh7Ilo9roFkQCfb6s2
uD636mHDGdFisml1THNOMUQ=
=TsNf
-END PGP SIGNATURE-


=
Instruções para entrar na lista, sair da lista e usar a lista em
http://www.mat.puc-rio.br/~nicolau/olimp/obm-l.html
=


Re: [obm-l] racional entre dois iracionais!!

2004-06-18 Por tôpico Fábio Dias Moreira
-BEGIN PGP SIGNED MESSAGE-
Hash: SHA1

Thiago Ferraiol [EMAIL PROTECTED] said:
 Desculpe minha ignorância mas será que você poderia ser mais claro???

 Não consegui ver como demonstrar a partir de tal sequencia!!!
 [...]

Uma propriedade fundamental de todos os números reais é que eles podem ser tão 
bem aproximados por racionais quanto queiramos. Logo, se o seu intervalo é 
[a, b], existe um certo r  a que aproxima a com erro menor que e, para todo 
e positivo. Em particular, existe um certo r tal que |r - a|  |b - a|. Mas 
como r  a, isso implica r - a  b - a == r  b == a  r  b, com r 
racional.

[]s,

- -- 
Fábio Dias Moreira
-BEGIN PGP SIGNATURE-
Version: GnuPG v1.2.3 (GNU/Linux)

iD8DBQFA0vh+alOQFrvzGQoRAoc0AJ0cgBty6LrwFrE2YJJ2PdNJi4WNEQCfbQRI
u9bmEnhHG7x/g3g/gSco8VM=
=S+LM
-END PGP SIGNATURE-


=
Instruções para entrar na lista, sair da lista e usar a lista em
http://www.mat.puc-rio.br/~nicolau/olimp/obm-l.html
=


Re: [obm-l] racional entre dois iracionais!!

2004-06-17 Por tôpico Fábio Dias Moreira
-BEGIN PGP SIGNED MESSAGE-
Hash: SHA1

Thiago Ferraiol [EMAIL PROTECTED] said:
 Pessoal...

 Dado dois números irracionais, como mostrar que sempre existe (ou não
 existe) um numero racional entre eles???
 [...]

Considere uma seqüência (r_1, r_2, ...) de aproximações racionais por excesso 
do extremo inferior.

[]s,

- -- 
Fábio Dias Moreira
-BEGIN PGP SIGNATURE-
Version: GnuPG v1.2.3 (GNU/Linux)

iD8DBQFA0nM8alOQFrvzGQoRAgG0AJ9U/RgO1VbIGarm7xtMJ+bPli5eUACdHGzj
YJdVaXS3S+nCwNbxSpvK/Dk=
=Thbs
-END PGP SIGNATURE-


=
Instruções para entrar na lista, sair da lista e usar a lista em
http://www.mat.puc-rio.br/~nicolau/olimp/obm-l.html
=


Re: [obm-l] CADEIAS DE MARKOV!

2004-06-16 Por tôpico Fábio Dias Moreira
-BEGIN PGP SIGNED MESSAGE-
Hash: SHA1

[EMAIL PROTECTED] said:
 [...]
 Os hábitos de fumar de um homem são como segue. Se ele fuma cigarros com
 filtro numa semana, ele muda para cigarros sem filtro na semana seguinte
 com probabilidade 0,2. Por outro lado, a probabilidade de que ele fume
 cigarros sem filtro, em duas semanas seguidas é 0,7. A longo prazo, durante
 que parte do tempo ele fuma cigarros com filtro?
 [...]

A matriz de transição da cadeia é 1/10*[8 3; 2 7], que tem apenas um autovetor 
com autovalor associado igual a 1, que é t*(3, 2). Como a soma das 
probabilidades tem que valer 1, t vale 5, logo ele fuma cigarros com filtro 
com probabilidade 3/5 a longo prazo.

[]s,

- -- 
Fábio Dias Moreira
-BEGIN PGP SIGNATURE-
Version: GnuPG v1.2.3 (GNU/Linux)

iD8DBQFA0OPJalOQFrvzGQoRAvgHAJ9lKn7LfmkOrHAG6nBJLJF0CMbpNwCfSqaW
HWD7M5VPbrVDHoVObC15Fe4=
=gnoC
-END PGP SIGNATURE-


=
Instruções para entrar na lista, sair da lista e usar a lista em
http://www.mat.puc-rio.br/~nicolau/olimp/obm-l.html
=


Re: [obm-l] residuos quadráticos (ajuda!!!)

2004-06-14 Por tôpico Fábio Dias Moreira
-BEGIN PGP SIGNED MESSAGE-
Hash: SHA1

Thiago Ferraiol [EMAIL PROTECTED] said:
 Pessoal...

 Estou com uma dúvida para resolver congruencias do tipo x^2 = a (mod p)
 onde p é primo impar
 [...]
 No meio de um exercicio apareceu a seguinte congruência d^2 = 56 (mod
 61)... alguém poderia me ajudar
 [...]

As únicas propriedades que você precisa saber para calcular um símbolo de 
Legendre são as seguintes:

* (ab/p) = (a/p)(b/p) se (a, b) = 1;
* (a^2*b/p) = (b/p);
* (p/q)(q/p) = (-1)^[(p-1)(q-1)/4] se p e q são primos ímpares;
* (-1/p) = (-1)^[(p-1)/2] se p é primo ímpar;
* (2/p) = (-1)^[(p^2-1)/8] se p é primo ímpar;

Note que 56 = 2^2*14. Então

(56/61) = (14/61) = (2/61)*(7/61) = (-1)*(61/7) = (-1)*(5/7) = -(7/5) = -(2/5) 
= 1.

Se você quiser ver demosntrações dos fatos acima, o livro do Plínio de Teoria 
dos Números, editado pela SBM, é uma boa referência.

[]s,

- -- 
Fábio Dias Moreira
-BEGIN PGP SIGNATURE-
Version: GnuPG v1.2.3 (GNU/Linux)

iD8DBQFAzn1BalOQFrvzGQoRAjDYAJ4uLHCrRJmVO/ycntCVvzvFWClr3QCfagRy
o41Vi2y2wNjozmkHf783dZo=
=QD0D
-END PGP SIGNATURE-


=
Instruções para entrar na lista, sair da lista e usar a lista em
http://www.mat.puc-rio.br/~nicolau/olimp/obm-l.html
=


Re: [obm-l] Re: Ajuda²

2004-06-10 Por tôpico Fábio Dias Moreira
-BEGIN PGP SIGNED MESSAGE-
Hash: SHA1

Wallace Martins [EMAIL PROTECTED] said:
 Fábio Dias Moreira escreve:
 [...]
 [EMAIL PROTECTED] said:
 Queria ajuda da turma em algumas questões:

 1) O produto das raízes do seguinte sistema

 {X elevado a Logy + Y elevado a Logx = 200
 {raíz de X elevado a Logy multiplicado por Y elevado a Logx = y

 a) 1 b) 1000 c) 100 d )  10
 [...]
 [...]
 y = 1000
 x = 1/10

 x*y = 100.
 [...]

 Fábio,

   por que x = 10^{-1} e não x = 10^{2/3}?
 [...]

Porque a minha conta está errada -- obrigado pela correção.

[]s,

- -- 
Fábio Dias Moreira
-BEGIN PGP SIGNATURE-
Version: GnuPG v1.2.3 (GNU/Linux)

iD8DBQFAyIjYalOQFrvzGQoRAtBPAJ9UJJ360mcL5aDOMLuAqvKbjF2AKgCdGHYk
1UkX+adR5+ByJXW0StR4Xlo=
=7YnO
-END PGP SIGNATURE-


=
Instruções para entrar na lista, sair da lista e usar a lista em
http://www.mat.puc-rio.br/~nicolau/olimp/obm-l.html
=


Re: [obm-l] Ajuda²

2004-06-09 Por tôpico Fábio Dias Moreira
-BEGIN PGP SIGNED MESSAGE-
Hash: SHA1

[EMAIL PROTECTED] said:
 Queria ajuda da turma em algumas questões:

 1) O produto das raízes do seguinte sistema

 {X elevado a Logy + Y elevado a Logx = 200
 {raíz de X elevado a Logy multiplicado por Y elevado a Logx = y

 a) 1 b) 1000 c) 100 d )  10
 [...]

x^log(y)+y^log(x) = 200
exp(log(x) * log(y)) + exp(log(y) * log(x)) = 200
log(x)*log(y) = log(100)

sqrt(x)^log(y)*y^log(x)=y
exp(log(x)*log(y)/2+log(y)*log(x)) = y
y = exp(log(10) + log(100))
y = 1000
x = 1/10

x*y = 100.

[]s,

- -- 
Fábio Dias Moreira
-BEGIN PGP SIGNATURE-
Version: GnuPG v1.2.3 (GNU/Linux)

iD8DBQFAx2DLalOQFrvzGQoRAp9qAJwP+4eGJSzW+sB696BpoFXEznKeTgCcDo/r
lcAQPmtiQOyRc5kU3Hhtd2s=
=cPUE
-END PGP SIGNATURE-


=
Instruções para entrar na lista, sair da lista e usar a lista em
http://www.mat.puc-rio.br/~nicolau/olimp/obm-l.html
=


Re: [obm-l] Re: [obm-l] função monótona

2004-06-05 Por tôpico Fábio Dias Moreira
-BEGIN PGP SIGNED MESSAGE-
Hash: SHA1

Osvaldo [EMAIL PROTECTED] said:
 Acredito que seja um dos tipos de funçoes abaixo:

 Estritamente crescente;
 Estritamente decrescente;
 Crescente;
 Decrescene;

 Os dois primeiros tipos de funçoes monotonas acima tem
 a prop. de que a derivada de primeira ordem nunca se
 anula e os dois restantes que ela nao é nula em todo
 intervalo, porem podendo anular se em um subconjunto
 do domínio.
 [...]

Isso se existir uma derivada...

[]s,

- -- 
Fábio Dias Moreira
-BEGIN PGP SIGNATURE-
Version: GnuPG v1.2.3 (GNU/Linux)

iD8DBQFAwmPZalOQFrvzGQoRAiQpAJ42omajTAASUS5RGweCsCfmbflgdgCeKplY
NwLZgculuoTTjeUzrks7FsI=
=DjqD
-END PGP SIGNATURE-


=
Instruções para entrar na lista, sair da lista e usar a lista em
http://www.mat.puc-rio.br/~nicolau/olimp/obm-l.html
=


Re: [obm-l] Aritmética

2004-06-04 Por tôpico Fábio Dias Moreira
-BEGIN PGP SIGNED MESSAGE-
Hash: SHA1

aryqueirozq [EMAIL PROTECTED] said:
Poderiam me ajudar nesta questão?

 Um lojista está disposto a vender um tênis de três
 formas:

 I - R$100,00 à vista hoje
 II - R$125,00 daqui a um mês
 III - Duas parcelas de R$60,00, uma hoje e outra daqui
 a um mês.
 Se você sabe que a inflação é de 30% ao mês e que 90%
 desta é repassado ao seu salário, qual é a melhor opção
 de compra?
 [...]

Isso quer dizer que o dinheiro vale 27% ao mês.

Transportando todos os valores para a época atual:

I - 100 reais
II - 125/1,27 = 98,43 reais
III - 60 + 60/1,27 = 107,24 reais

A melhor opção é a II.

[]s,

- -- 
Fábio Dias Moreira
-BEGIN PGP SIGNATURE-
Version: GnuPG v1.2.3 (GNU/Linux)

iD8DBQFAwR2balOQFrvzGQoRAp1AAJ0SlQ1Qko5qtuyFK1MUELFCnqS2vACffms1
p1z0jZRbYibX462Zq9cSM88=
=b+bZ
-END PGP SIGNATURE-


=
Instruções para entrar na lista, sair da lista e usar a lista em
http://www.mat.puc-rio.br/~nicolau/olimp/obm-l.html
=


Re: [obm-l] Números Interessantes

2004-06-03 Por tôpico Fábio Dias Moreira
-BEGIN PGP SIGNED MESSAGE-
Hash: SHA1

Qwert Smith [EMAIL PROTECTED] said:
   Questão: Quantos são os números com 10 algarismo diferentes entre
  si e divisível por 1.
   Dizer que eles estão incluídos entre os números interessantes está
  correto?

 Se um numero de 10 algarismos tem todos os algarismos distintos entao
 eh multiplo de 9 (0+1+2+3+4+5+6+7+8+9=45,4+5=9)

 Queremos entao os multiplos de 9 que tem 10 algarismos, todos eles
 distintos

 todos os multiplo de 9 no intervalo [1023456789,9876543210] sao
 da forma 9 * ABCDE com A,B,C,D e E inteiros =9

 A pergunta entao passa a ser quantos numeros de 5 algarismos  quando
 multiplicaods por 9 resultam em numeros de 10 algarismos distintos

 ABCDE*9=ABCDE*(10-1)=ABCDE*10-ABCDE=
 ABCD(E-1)(9-A)(9-B)(9-C)(9-D)(10-E), fazendo E-1=F para
 ficar mais legivel (se eh ki isso eh possivel)
 ABCDE*9=ABCDF(9-A)(9-B)(9-C)(9-D)(9-F)
 [...]

E-1 pode não ser um dígito (neste caso, é fácil ver que o número vai ter dois 
dígitos 0, mas isso é importante para fechar a demonstração).

[]s,

- -- 
Fábio Dias Moreira
-BEGIN PGP SIGNATURE-
Version: GnuPG v1.2.3 (GNU/Linux)

iD8DBQFAv8sdalOQFrvzGQoRAtJdAJ0RaC6t0e1+zJj8jgLzoo41IK/M5gCgq6A4
Xx5j/URUkzEH7w/1+n0KLvc=
=4fX5
-END PGP SIGNATURE-


=
Instruções para entrar na lista, sair da lista e usar a lista em
http://www.mat.puc-rio.br/~nicolau/olimp/obm-l.html
=


Re: [obm-l] VALOR ESPERADO!

2004-06-02 Por tôpico Fábio Dias Moreira
-BEGIN PGP SIGNED MESSAGE-
Hash: SHA1

[EMAIL PROTECTED] said:
 [...]
 A probabilidade de a equipe A vencer qualquer jogo é 1/2. A e B disputam
 entre si um torneio. A primeira equipe que conseguir vencer dois jogos em
 seguida ou um total de três jogos vence o torneio. Determinar o número
 esperado de jogos do torneio.
 [...]

Dados insuficientes, pois não se sabe a probabilidade de que A e B empatem.

[]s,

- -- 
Fábio Dias Moreira
-BEGIN PGP SIGNATURE-
Version: GnuPG v1.2.3 (GNU/Linux)

iD8DBQFAvpHPalOQFrvzGQoRAl3gAKCCtpBZZ+oHEFsFc6z8KNX3UrWWygCbBMA4
cimF0eR//WIE0eJ/9ETrSas=
=7EpO
-END PGP SIGNATURE-


=
Instruções para entrar na lista, sair da lista e usar a lista em
http://www.mat.puc-rio.br/~nicolau/olimp/obm-l.html
=


Re: [obm-l] Cosseno nao eh polinomio (2)

2004-05-31 Por tôpico Fábio Dias Moreira
-BEGIN PGP SIGNED MESSAGE-
Hash: SHA1

Claudio Buffara [EMAIL PROTECTED] said:
 Uma versao um pouco mais dificil:

 Sejam a e b numeros reais com a  b.
 Prove que F:[a,b] - R dada por F(x) = cos(x)
 nao eh uma funcao polinomial.
 [...]

F = F = F^(4k) = F, mas se F é uma função polinomial de grau n, então 
F^(n+1) = 0.  Mas tomando 4k = n+1, F^(4k) = F = 0, logo F é identicamente 
nula, absurdo.

[]s,

- -- 
Fábio Dias Moreira
-BEGIN PGP SIGNATURE-
Version: GnuPG v1.2.3 (GNU/Linux)

iD8DBQFAvAc9alOQFrvzGQoRAjm/AJ9Ah/0BIP04mSHIWCJocP6ZHMoFJACgqknp
+iEe7Grgty5DwhXM78IbWlk=
=Xuot
-END PGP SIGNATURE-


=
Instruções para entrar na lista, sair da lista e usar a lista em
http://www.mat.puc-rio.br/~nicolau/olimp/obm-l.html
=


Re: [obm-l] Geometria Espacial

2004-05-30 Por tôpico Fábio Dias Moreira
-BEGIN PGP SIGNED MESSAGE-
Hash: SHA1

Fabio Contreiras [EMAIL PROTECTED] said:
 Acabei de sair de uma prova no qual me deparei com a questão :

 A área da superfície lateral de um cone equilátero inscrito numa esfera de
 raio R é ?


 Gostaria de uma solução plausível para que o resultado dê [ ( pi R.R
 sqrt(3) ) / 2 ] !

 ps. achei [ (  pi . 3.R.R ) / 2 ]
 [...]

Não existe uma solução plausível porquê a resposta é essa mesmo (3*pi/2*R^2).

[]s,

- --
Fábio ctg \pi Dias Moreira
-BEGIN PGP SIGNATURE-
Version: GnuPG v1.2.3 (GNU/Linux)

iD8DBQFAuiG2alOQFrvzGQoRAm7CAKCRL5VaxCHUdfUlFEw+3qT5Qy/MkACg33ud
exZ0MGN20tppz698Epo1yUE=
=hdQJ
-END PGP SIGNATURE-


=
Instruções para entrar na lista, sair da lista e usar a lista em
http://www.mat.puc-rio.br/~nicolau/olimp/obm-l.html
=


Re: [obm-l] ITA 73

2004-05-30 Por tôpico Fábio Dias Moreira
-BEGIN PGP SIGNED MESSAGE-
Hash: SHA1

[EMAIL PROTECTED] said:
 oi pessoal! Sou o Rafael Lima aluno do Sistema Elite de Ensino- RJ e queria
 q vcs me ajuda-se a Fazer essa questão: Seja a equação do 4°
 x^4+qx^3+rx^2+sx+t=0 onde q,r,s,t são números racionais não nulos tais que
 l,m,n,p são raízes reais dessa equação. o valor de l/mnp + m/lnp + n/lmp +
 p/lmn = ???
 [...]

Isso é (l^2 + m^2 + n^2 + p^2)/(lmnp).

[]s,

- -- 
Fábio Dias Moreira
-BEGIN PGP SIGNATURE-
Version: GnuPG v1.2.3 (GNU/Linux)

iD8DBQFAunBxalOQFrvzGQoRApM2AKDl7iEhhg2/OWyFa+w8Zu2e4splPACgrbn1
8JSLsCMbn+lmT+1XP03N71w=
=jeX2
-END PGP SIGNATURE-


=
Instruções para entrar na lista, sair da lista e usar a lista em
http://www.mat.puc-rio.br/~nicolau/olimp/obm-l.html
=


Re: [obm-l] ITA-95

2004-05-19 Por tôpico Fábio Dias Moreira
-BEGIN PGP SIGNED MESSAGE-
Hash: SHA1

Márcio Barbado Jr. [EMAIL PROTECTED] said:
   Senhores (as)

   Estava analisando o material do cursinho Etapa, no que se refere a
 resolucao da prova de matemática do vestibular do ITA (ano 1995). Tenho ca
 comigo duvidas acerca da veracidade das afirmações contidas naquele
 material, entretanto posso ter esquecido algum teorema que venha a me
 calar. Vejam se podem me ajudar, aqui vai o enunciado (logo em seguida
 comentarei onde estou tropeçando). E a questao numero 9:
 [...]
   Daí vem minha duvida, que e a segunda afirmação ali contida. Sem
 mais nem menos, o texto afirma que, SE 5^(1/2) É RAIZ, ENTAO -5^(1/2)
 TAMBEM E.
 Deste ponto adiante, a apostila usa a primeira relação de GIRARD e
 voila!...

   Vejam bem: De fato -5^(1/2) será raiz! O problema e a afirmação de
 que se lancou mão.

   De forma bastante clara, minha duvida e: A ultima afirmação esta
 certa? Por que? Ou por que nao?
 [...]

Sim, ela está certa. De uma maneira geral, se um polinômio com coeficientes 
inteiros admite a raiz a + sqrt(b), então ele também admite a - sqrt(b) como 
raiz. A demonstração é exatamente a mesma que é utilizada para demonstrar que 
x + yi é raiz = x - yi é raiz (de fato, este é um caso particular do caso 
acima, onde b = -y^2).

[]s,

- -- 
Fábio ctg \pi Dias Moreira
-BEGIN PGP SIGNATURE-
Version: GnuPG v1.2.3 (GNU/Linux)

iD8DBQFAq3sHalOQFrvzGQoRAiQPAKDeDyymuMj2ydnO+TWSKkCrXB2PpACgszH9
ft7A6e4Lxg3S5kBD2Cqk4D4=
=Hgv+
-END PGP SIGNATURE-


=
Instruções para entrar na lista, sair da lista e usar a lista em
http://www.mat.puc-rio.br/~nicolau/olimp/obm-l.html
=


Re: [obm-l] Conjectura de Goldbach

2004-04-28 Por tôpico Fábio Dias Moreira
-BEGIN PGP SIGNED MESSAGE-
Hash: SHA1

Everton A. Ramos (www.bs2.com.br) [EMAIL PROTECTED] said:
 Boa noite...

 Todo número par é a soma de dois números ímpares

 ???

 Sedo X um número par... (X - 1) será ímpar... então (X - 1) + 1 = X

 1 é ímpar, então... ???

 Porque isso é tão desafiante?
 [...]

Porquê não é essa a conjectura de Goldbach -- segundo ela, todo par pode ser 
escrito como soma de dois ***PRIMOS***.

[]s,

- -- 
Fábio ctg \pi Dias Moreira
-BEGIN PGP SIGNATURE-
Version: GnuPG v1.2.3 (GNU/Linux)

iD8DBQFAkGaualOQFrvzGQoRAh3NAJ9q0U472IGP2Qqvg2dF5ztqHJ+FMwCgy4cZ
6dN8lcjDx3tMNw7kAD9o0EY=
=jjEb
-END PGP SIGNATURE-


=
Instruções para entrar na lista, sair da lista e usar a lista em
http://www.mat.puc-rio.br/~nicolau/olimp/obm-l.html
=


Re: [obm-l] duvida basica: mdc e mmc, conjunto

2004-04-14 Por tôpico Fábio Dias Moreira
-BEGIN PGP SIGNED MESSAGE-
Hash: SHA1

[EMAIL PROTECTED] [EMAIL PROTECTED] said:
 alguem sabe me dizer pq mdc(a,b) x mmc(a,b) = a x b?

 existe alguma explicacao/demonstracao?
 [...]

Prove, inicialmente, que max{a, b} + min{a, b} = a + b. Depois disso, pense em 
como achar o mmc e o mdc de a e b conhecendo as suas formas fatoradas.

 [...]

 e em

 n(a) + n(b) = n(a)+ n(b) - n(a inter b)

 nos conjuntos, eu consigo o entender pq (os elementos da intersecao sao
 somados duas vezes) intuitivamente. Porem eu estou estudando por um livro
 aki q pede pra provar. Alguem poderia me ajudar?
 [...]

Você quer dizer |A U B| = |A| + |B| + |A inter B|?

Começe calculando o número de elementos de |A - B| em função de |A| e |A inter 
B|.

[]s,

- -- 
Fábio ctg \pi Dias Moreira
-BEGIN PGP SIGNATURE-
Version: GnuPG v1.2.3 (GNU/Linux)

iD8DBQFAfVloalOQFrvzGQoRAt9eAKCBZCDcujV1ZjFi1TgohsPyi94KZwCfVpgq
Xl4PHdoBRpwr7tQrjLFMyF8=
=k7+z
-END PGP SIGNATURE-


=
Instruções para entrar na lista, sair da lista e usar a lista em
http://www.mat.puc-rio.br/~nicolau/olimp/obm-l.html
=


Re: [obm-l] RSA QUEBRA

2004-04-13 Por tôpico Fábio Dias Moreira
-BEGIN PGP SIGNED MESSAGE-
Hash: SHA1

[EMAIL PROTECTED] said:
 Caros amigos, há um bom tempo a criptografia RSA me chama atenção. Desde
 então, venho estudando sua matemática. Hoje, possuo um vasto trabalho
 realizado na possível quebra de qq RSA, com a montagem de um algoritmo
 geral ou com pequenas modificaçãoes. Gostaria de ter a opinião da lista
 sobre a possibilidade de quebrar RSA, tão facilmente como somar 2 e 2.
 Espero em breve, mostrar meus resultados e poder discutí-los com todos
 voces. Sei da importância q RSA tem para a Internet e segurança de dados.
 Opinem. Abraço.
 [...]

Talvez você goste de saber que US$675.000,00 estão à procura de um novo dono:

http://www.rsasecurity.com/rsalabs/challenges/factoring/numbers.html

[]s,

- -- 
Fábio ctg \pi Dias Moreira
-BEGIN PGP SIGNATURE-
Version: GnuPG v1.2.3 (GNU/Linux)

iD8DBQFAfKwUalOQFrvzGQoRAmZKAJ9j292p0ivzlE8u6qjm64WEyWFMpACgg4gq
bI/cRQPE6Yjk8+smpZh7okM=
=BLy8
-END PGP SIGNATURE-


=
Instruções para entrar na lista, sair da lista e usar a lista em
http://www.mat.puc-rio.br/~nicolau/olimp/obm-l.html
=


Re: [obm-l] logaritmo

2004-04-04 Por tôpico Fábio Dias Moreira
-BEGIN PGP SIGNED MESSAGE-
Hash: SHA1

Guilherme Teles [EMAIL PROTECTED] said:
 Alguem sabe resolver essa:

 3^log1/3 2
 a base é 1/3 (1 sobre 3)
 [...]

Para poder simplificar o logaritmo com a exponencial, eu preciso que os dois 
tenham a mesma base. Por isso, eu faço uma conversão de base, para que 
apareça algum logaritmo com base 3:

log[1/3](2) = log[1/3](3^log[3](2)) = log[3](2)*log[1/3](3). Mas eu sei que 
(1/3)^(-1) = 3, portanto log[1/3](3) = -1 = log[1/3](2) = -log[3](2).

Então

3^log[1/3](2) = 3^-log[3](2) = 1/(3^log[3](2)) = 1/2.

[]s,

- -- 
Fábio Dias Moreira
http://dias.moreira.nom.br/
-BEGIN PGP SIGNATURE-
Version: GnuPG v1.2.3 (GNU/Linux)

iD8DBQFAcIagalOQFrvzGQoRAjtEAJ42x3X6Z/gHKNRPhR/ND81Sg1KhRQCdFGPA
2GfUJfitwhdk1zUU46cCgRg=
=IxBO
-END PGP SIGNATURE-


=
Instruções para entrar na lista, sair da lista e usar a lista em
http://www.mat.puc-rio.br/~nicolau/olimp/obm-l.html
=


Re: [obm-l] O JOGO DE RENCONTRE! - CORREÇÃO

2004-04-04 Por tôpico Fábio Dias Moreira
-BEGIN PGP SIGNED MESSAGE-
Hash: SHA1

[EMAIL PROTECTED] said:
 COrreçãozinha, em que eu troquei a ordem para não confundir sobre de que é
 o fatorial:

 Mais tarde eu espero conseguir explicar, mas creio que a solução é

 S = (1/n!)*[ 1 + S_a + S_b ], em que

  S_a = Somatório de { A*(n!)/[ (n - 2A)!*2A ] }, com A variando de 1 até

 o inteiro menor ou igual a (n - 1)/2

  S_b = Somatório de { B*(n!)/[ (n - 2B - 1)!*(2B + 1) ] }, com B
  variando

 de 1 até o inteiro menor ou igual a (n - 2)/2

 Claro que, para incorporar S_a, devemos ter n =3 , e, para incorporar S_b,
 devemos ter n = 4.
 [...]

Para n=5, S_a = 1*5!/(3!*2) + 2*5!/(1!*4) = 10 + 60 = 70, S_b = 1*5!/(2!*3) = 
20, logo existiriam 70+20+1 = 91 permutações com rencontres. Mas, na 
realidade, só há 76:

01234 01243 01324 01342 01423 01432 02134 02143 02314 02341
02413 02431 03124 03142 03214 03241 03412 03421 04123 04132
04213 04231 04312 04321 10234 10243 10324 10432 12034 12304
12430 13024 13204 13240 14032 14203 14230 20134 20314 20431
21034 21043 21304 21340 21403 21430 23014 23104 24031 24130
30124 30214 30241 31024 31042 31204 31240 31402 31420 32014
32104 34201 34210 40132 40213 40231 41023 41032 41203 41230
41302 41320 42031 42130 43201 43210

(Eu usei um programinha para gerar a lista acima.)

[]s,

- -- 
Fábio Dias Moreira
http://dias.moreira.nom.br/
-BEGIN PGP SIGNATURE-
Version: GnuPG v1.2.3 (GNU/Linux)

iD8DBQFAcMCqalOQFrvzGQoRArbzAKDiTpKoTAzjhyJkCOabtM8uLIRm4gCfVn+W
Et1H9PUYEp0d6WU5JOD+r5Q=
=HDC4
-END PGP SIGNATURE-


=
Instruções para entrar na lista, sair da lista e usar a lista em
http://www.mat.puc-rio.br/~nicolau/olimp/obm-l.html
=


Re: [obm-l] dúvida

2004-04-03 Por tôpico Fábio Dias Moreira
-BEGIN PGP SIGNED MESSAGE-
Hash: SHA1

TSD [EMAIL PROTECTED] said:
 um determinado fio é constituído de um material que, quando preso a dois
 pontos distantes um do outro de 20m e ambos a 13m do solo, toma a forma de
 uma parábola, estando o ponto mais baixo do fio a 3m do solo. Assinale a
 alternativa que corresponde à parábola no sistema de coordenadas
 cartesianas XOY, onde o eixo OY contém o ponto mais baixo do fio e o eixo
 OX está sobre o solo.
 [...]

Inicialmente, note que fios suspensos *não* formam parábolas, mas sim 
catenárias, que são coisas parecidas com o gráfico de (e^x + e^-x)/2.

Pedantismos físicos à parte, note que se os pontos de apoio são os pontos (10, 
13) e (-10, 13), então o ponto mais baixo é o ponto (0, 3). Associe à 
parábola uma função f(x) tal que todos os pontos da parábola são da forma (x, 
f(x)). Portanto, f(10) = f(-10) = 13 e f(0) = 3.

Obviamente, f é quadrática. Considere g(x) = f(x) - 13. Então 10 e -10 são 
dois zeros de g, logo g(x) = a*(x-10)*(x+10), onde a é um real. Como g(0) = 
- -10, a*(-10)*10 = -10 = a = 1/10. Logo g(x) = (x-10)*(x+10)/10, logo f(x) = 
(x-10)*(x+10)/10 + 13. Como as alternativas da questão não chegaram aqui, 
essa é a melhor resposta que eu posso dar.

[]s,

- -- 
Fábio Dias Moreira
http://dias.moreira.nom.br/
-BEGIN PGP SIGNATURE-
Version: GnuPG v1.2.3 (GNU/Linux)

iD8DBQFAb1YbalOQFrvzGQoRAmytAKDAnMNnuzn97j+I85/F8k+fOiM2xACfbeon
dRZpXPhtXJ8ltTc/tINuXZQ=
=bpms
-END PGP SIGNATURE-


=
Instruções para entrar na lista, sair da lista e usar a lista em
http://www.mat.puc-rio.br/~nicolau/olimp/obm-l.html
=


Re: [obm-l] RE: probabilidade-reencontre

2004-04-03 Por tôpico Fábio Dias Moreira
-BEGIN PGP SIGNED MESSAGE-
Hash: SHA1

André Zimmermann [EMAIL PROTECTED] said:
 Também surgiu-me dúvida.

 Uma urna contém n bilhetes numerados 1, 2, , n. Extraem-se os bilhetes
 de um a um sem reposição, se aparecer o bilhete numerado r na r-ésima
 extração, designa-se isto como um match ou um rencontre. Determinar a
 probabilidade de ter pelo menos um rencontre!

 Raciocinio meu:

A probabilidade de fazer um reencontro é de 1/n a cada rodada pois:

A cada rodada, o número de bilhetes na urna será de (n-r). A
 probabilidade de se retirar o número da vez será 1/(n-r) e a probabilidade
 do número procurado ainda estar na urna é igual a (n-r)/n.

Então [1/(n-r)]x[(n-r)/n] =   1/n.

Já que a urna contém n bilhetes, repetiremos a operação de retirada n
 vezes.

A probabilidade de se fazer pelo menos um reencontre é n x 1/n. ou seja,
 100%. ??
 [...]

Os eventos não são independentes, logo a análise não é tão simples assim.

[]s,

- -- 
Fábio Dias Moreira
http://dias.moreira.nom.br/
-BEGIN PGP SIGNATURE-
Version: GnuPG v1.2.3 (GNU/Linux)

iD8DBQFAb2P1alOQFrvzGQoRAnTxAKDhLFwkphKun6Ps0EYTX23UQnaEMACfT74z
zA/X88FNz299A2j+koqYhmY=
=oUTL
-END PGP SIGNATURE-


=
Instruções para entrar na lista, sair da lista e usar a lista em
http://www.mat.puc-rio.br/~nicolau/olimp/obm-l.html
=


Re: [obm-l] Logaritmos

2004-04-02 Por tôpico Fábio Dias Moreira
-BEGIN PGP SIGNED MESSAGE-
Hash: SHA1

[EMAIL PROTECTED] said:
 Prove que, para todo inteiro n maior que 1 e para todo x diferente de zero,
 com x maior que -1, tem-se:

  (1+x)^n  (1+nx)
 [...]

Para n=2 a desigualdade é obviamente verdadeira. Suponha que ela é verdadeira 
para n. Então

(1+x)^(n+1) = (1+x)^n*(1+x)  (1+nx)(1+x) = 1 + nx + x + x^2  1 + (n+1)x.

[]s,

- -- 
Fábio Dias Moreira
http://dias.moreira.nom.br/
-BEGIN PGP SIGNATURE-
Version: GnuPG v1.2.3 (GNU/Linux)

iD8DBQFAbe/6alOQFrvzGQoRAh/EAKCBB1XwUynps4iTe5ykLky4VTIdowCeIZBw
Z0KdtSt3PAiMClDeJ+RjIAg=
=M/XC
-END PGP SIGNATURE-


=
Instruções para entrar na lista, sair da lista e usar a lista em
http://www.mat.puc-rio.br/~nicolau/olimp/obm-l.html
=


Re: [obm-l] Números primos

2004-03-30 Por tôpico Fábio Dias Moreira
-BEGIN PGP SIGNED MESSAGE-
Hash: SHA1

Fábio Bernardo [EMAIL PROTECTED] said:
 Pessoal, tô enrolado nesse. Ajudem-me por favor.

 Sejam x e y dois números primos. Determine quantos pares ordenados (x,y)
 existem, tal que x+y = 497.
 [...]

Se dois números inteiros têm soma ímpar, o que se pode afirmar quanto à 
paridade deles?

[]s,

- -- 
Fábio Dias Moreira
-BEGIN PGP SIGNATURE-
Version: GnuPG v1.2.3 (GNU/Linux)

iD8DBQFAai6QalOQFrvzGQoRAoMHAJ9s1kxyh1/hLRKIzAhBWHfoheD/cACfWK3o
OIRey25DP/uK6jYJalINLbA=
=ZIg1
-END PGP SIGNATURE-


=
Instruções para entrar na lista, sair da lista e usar a lista em
http://www.mat.puc-rio.br/~nicolau/olimp/obm-l.html
=


Re: [obm-l] funcao geradora de momentos

2004-03-25 Por tôpico Fábio Dias Moreira
-BEGIN PGP SIGNED MESSAGE-
Hash: SHA1

niski [EMAIL PROTECTED] said:
 Pessoal, infelizmente não consigo uma boa referencia de estatistica que
 aborde esse assunto(alguem conhece alguma que nao seja o livro do
 Ross?(este esta sempre alugado na minha biblioteca))

 Vamos a minha pergunta
 Eu sei que a definição (para o caso discreto) é
 m(t) = E[exp(tX)] = Somatorio(x, ,) exp(tx)*f(x)


 Ai me aparece essa pergunta
 Sejam X_{0},X_{1} v.a. independentes, X_{0} ~ Poisson(1), X_{1} ~
 Poisson(2),
 e seja Y uma v.a. Bernoulli(p), independente de X_{0},X_{1}. Calcule a
 funcao
 geradora de momentos de X_{Y}

 E agora? Como eu aplico a definicao?
 [...]

A função de dstribuição de probabilidade de X_Y é (1-p)*f_0+p*f_1, onde f_0 e 
f_1 são as funções de distribuição de probabilidade de X_0 e X_1.

[]s,

- -- 
Fábio ctg \pi Dias Moreira
-BEGIN PGP SIGNATURE-
Version: GnuPG v1.2.3 (GNU/Linux)

iD8DBQFAY3KEalOQFrvzGQoRAlwnAKCvW63ufW3piPafSbtuHuDXRAwR4QCgqRiO
jfqE+8W+a4Xj1zsWeMdNBUg=
=NZGC
-END PGP SIGNATURE-


=
Instruções para entrar na lista, sair da lista e usar a lista em
http://www.mat.puc-rio.br/~nicolau/olimp/obm-l.html
=


Re: [obm-l] problema estatistico - trigonometrico

2004-03-21 Por tôpico Fábio Dias Moreira
-BEGIN PGP SIGNED MESSAGE-
Hash: SHA1

niski [EMAIL PROTECTED] said:
 É dado que X ~ U[-pi, pi] e considere as v.a Y = senX , Z = cosX
 Pergunta: As v.a Y,Z são independentes?
 [...]

Não. A probabilidade de que Y e Z sejam ambas menores que -0.8, por exemplo, é 
obviamente zero, mas a probabilidade de que Y seja menor que -0.8 é positiva 
(idem para Z).

[]s,

- -- 
Fábio ctg \pi Dias Moreira
-BEGIN PGP SIGNATURE-
Version: GnuPG v1.2.3 (GNU/Linux)

iD8DBQFAXhBmalOQFrvzGQoRAs8AAKDZrlrQDKJX2mDyI+GGttChDydBWwCfdkXC
F7tSNvtYjzmuWqEDCSD0PrU=
=PZWT
-END PGP SIGNATURE-


=
Instruções para entrar na lista, sair da lista e usar a lista em
http://www.mat.puc-rio.br/~nicolau/olimp/obm-l.html
=


Re: [obm-l] P.A

2004-03-21 Por tôpico Fábio Dias Moreira
-BEGIN PGP SIGNED MESSAGE-
Hash: SHA1

elton francisco ferreira [EMAIL PROTECTED] said:
  1 - Determine a P.A em que se verificam as
 propriedades seguintes:

 a5 + a8 = 130 e a4 + a10 = 140
 [...]

a_4 + a_10 = a_5 + a_9.

 [...]
 2 - Qual é a P.A finita em que o primeiro termo é 8, o
 último termo é 38, e o números de termos é igual a razão?
 [...]

Se a progressão é (a_0, a_1, ..., a_n), a_n = a_0 + n*r.

[]s,

- -- 
Fábio ctg \pi Dias Moreira
-BEGIN PGP SIGNATURE-
Version: GnuPG v1.2.3 (GNU/Linux)

iD8DBQFAXlIXalOQFrvzGQoRAqD8AKCxLiOEWwHIM+ijBEaWo/FijK1JkwCfShpp
/4qWh/bRbu+8fBFCgw1NR/A=
=YyP5
-END PGP SIGNATURE-


=
Instruções para entrar na lista, sair da lista e usar a lista em
http://www.mat.puc-rio.br/~nicolau/olimp/obm-l.html
=


Re: [obm-l] Duvidas

2004-03-20 Por tôpico Fábio Dias Moreira
-BEGIN PGP SIGNED MESSAGE-
Hash: SHA1

aryqueirozq [EMAIL PROTECTED] said:
 1)Determinar as coordenados dos vertices de um
 triângulo, sabendo que os
 pontos médios dos lados do triângulo são M(-2,1); N
 (5,2) e P(2,-3).
 [...]

Seja M = (A+B)/2; N = (B+C)/2; P = (C+A)/2. A que corresponde o ponto M+N-P?

 [...]
 2)Uma prova de múltlipa de escolha com 69 questões, foi
 corrigida da
 seguinte forma: o aluno ganhava 5 pontos para cada
 questao que acertava e
 perdia 1 ponto para a questão que ele errava ou deixava
 em branco.Se um
 aluno totalizou 210 pontos, o nº de questões que ele
 acertou é?
 A)25 B)30 C)35 D)40 E)45
 [...]

Tem certeza de que isto está certo? Como o número de pontos de cada questão é 
sempre ímpar, independente do fato da questão estar certa ou errada, e o 
número de questões na prova é ímpar, a pontuação final do aluno só pode ser 
um número ímpar (o que não é o caso de 210).

[]s,

- -- 
Fábio ctg \pi Dias Moreira
-BEGIN PGP SIGNATURE-
Version: GnuPG v1.2.3 (GNU/Linux)

iD8DBQFAXKldalOQFrvzGQoRAuQAAJ9DyOHsGsMtuen7l2uxEmxmV20kJACfddlu
0oahPft1JO/cDuUkP7kbuhk=
=jv+W
-END PGP SIGNATURE-


=
Instruções para entrar na lista, sair da lista e usar a lista em
http://www.mat.puc-rio.br/~nicolau/olimp/obm-l.html
=


Re: [obm-l] Duvida em somatorio

2004-03-18 Por tôpico Fábio Dias Moreira
-BEGIN PGP SIGNED MESSAGE-
Hash: SHA1

niski [EMAIL PROTECTED] said:
 Pessoal, alguem poderia mostrar como resolver esse somatorio por favor?
 (ele veio do calculo da esperança de 1/X onde X segue uma distribuicao
 geometrica)

 Somatorio[n=1 , +inf] [(1/n)*p*(1-p)^(n-1)]
 [...]

Calcule a série de Taylor de ln(1-x) em relação a x.

[]s,

- -- 
Fábio ctg \pi Dias Moreira
-BEGIN PGP SIGNATURE-
Version: GnuPG v1.2.3 (GNU/Linux)

iD8DBQFAWjN9alOQFrvzGQoRAqIbAJ9pSugZjZ+ZqOXjnKg7pkFXC6og5wCfdfyx
SA1SpLIBTQ+/HUiWHC3hvWM=
=t4cl
-END PGP SIGNATURE-


=
Instruções para entrar na lista, sair da lista e usar a lista em
http://www.mat.puc-rio.br/~nicolau/olimp/obm-l.html
=


Re: [obm-l] Divisibilidade

2004-03-14 Por tôpico Fábio Dias Moreira
-BEGIN PGP SIGNED MESSAGE-
Hash: SHA1

André Luiz Martins Guimarães Orsi [EMAIL PROTECTED] said:
 Olá,

Alguém conhece um critério de divisibilidade por 13, sem ser por
 congruência, tipo os critérios que existem para 2, 3, 5 ...
 [...]

http://www.mat.puc-rio.br/~nicolau/olimp/obm-l.200306/msg00796.html

Se você não estiver interessado na teoria por trás do critério, pule para o 
final da mensagem.

Um outro critério usa o fato de que 13|1001, logo x é divisível por 13 se e 
somente se a diferença entre os grupos de três algarismos de ordem par e os 
de prdem ímpar também for divisível por 13.

Por exemplo, no problema resolvido mentalmente pelo nosso colega Cláudio, 
temos que provar que 2^70 + 3^70 = 2503155504994422192936289397389273 é 
múltiplo de 13. Quebrando o número em grupos de 3, temos que

2^70 + 3^70 = 2.503.155.504.994.422.192.936.289.397.389.273

Esse número é divisível por treze se e somente se a diferença entre a soma das 
classes de ordem par (273+397+936+422+504+503) e as de ordem ímpar 
(389+289+192+994+155+2) for divisível por 13. Essa diferença vale |3035-2021| 
= 1014.

Pelo algoritmo do link acima, esse número é divisível por 13 se e somente se 
101 + 4*4 = 117 é divisível por 13, o que é verdade se e somente se 11 + 4*7 
= 39 = 3*13 é divisível por 13.

Ou então, note qe a diferença entre as ordens pares e as ímpares de 1014 é 13 
= 13*1.

[]s,

- -- 
Fábio ctg \pi Dias Moreira
-BEGIN PGP SIGNATURE-
Version: GnuPG v1.2.3 (GNU/Linux)

iD8DBQFAVSNNalOQFrvzGQoRAs0WAJ9KZpBpyPfhzKjaP72dc0YdsxgNRwCfXcH6
+CQXI/3ZYRff8Ct4WQmteCE=
=zGQn
-END PGP SIGNATURE-


=
Instruções para entrar na lista, sair da lista e usar a lista em
http://www.mat.puc-rio.br/~nicolau/olimp/obm-l.html
=


Re: [obm-l] Arctg de 1/Fibonacci

2004-03-12 Por tôpico Fábio Dias Moreira
-BEGIN PGP SIGNED MESSAGE-
Hash: SHA1

Ricardo Bittencourt [EMAIL PROTECTED] said:
 Claudio Buffara wrote:
  Calcule o valor da soma:
  SOMA(n = 1) arctg(1/F(n)),

   Não sei se facilita ou complica, mas e se você criasse
 uma sequencia c(n) onde c(n)=(F(n)+i) ? Nesse caso, o argumento
 desse número complexo é arctg(1/F(n)), e a somatória de todos
 eles é igual ao argumento do produtório de todos os c(n).
 [...]

Correto, mas aí o produtório dos c_n não converge. Se d_n = c_n/|c_n|, a idéia 
funciona, mas eu tenho a nítida impressão de que isso dá uma fórmula feia.

Eu sei provar que o arctg(1/F(3)) + arctg(1/F(5)) + arctg(1/F(7)) + ... vale 
pi/2. A soma dos termos de ordem par parece convergir para um valor próximo 
de, mas certamente menor que, pi/6.

[]s,

- -- 
Fábio ctg \pi Dias Moreira
-BEGIN PGP SIGNATURE-
Version: GnuPG v1.2.3 (GNU/Linux)

iD8DBQFAUmSoalOQFrvzGQoRAnXxAJ4kIPkdVqzXBPAySgGIXE4CGTmKSgCg3rHv
oBR7SRRbnKBTF/m6JoWX6ys=
=ogVT
-END PGP SIGNATURE-


=
Instruções para entrar na lista, sair da lista e usar a lista em
http://www.mat.puc-rio.br/~nicolau/olimp/obm-l.html
=


Re: [obm-l] Identidades de mdc

2004-03-07 Por tôpico Fábio Dias Moreira
-BEGIN PGP SIGNED MESSAGE-
Hash: SHA1

Rafael [EMAIL PROTECTED] said:
 Fábio,

 Primeiramente, deixe-me ver se entendi as suas notações.

 mdc(a,b) = (a,b) e mmc(a,b) = [a,b]
 [...]

Isso -- a notação para o mdc é muito comum, especialmente em um contexto de 
teoria dos números, onde não há pares ordenados envolvidos. A segunda notação 
é usada de vez em quando, mas é bem mais rara que a primeira, já que o mmc 
não é tão relevante assim do ponto de teoria dos números.

[]s,

- -- 
Fábio ctg \pi Dias Moreira
-BEGIN PGP SIGNATURE-
Version: GnuPG v1.2.3 (GNU/Linux)

iD8DBQFAS0KRalOQFrvzGQoRAuIRAJ9/iAkdAeRbY+XCHNx0xMrKs9ExjgCgj+xQ
nB8rCJMMK6H3Uw9LhCV21KA=
=Dc0w
-END PGP SIGNATURE-


=
Instruções para entrar na lista, sair da lista e usar a lista em
http://www.mat.puc-rio.br/~nicolau/olimp/obm-l.html
=


Re: [obm-l] Identidades de mdc

2004-03-06 Por tôpico Fábio Dias Moreira
-BEGIN PGP SIGNED MESSAGE-
Hash: SHA1

Rafael [EMAIL PROTECTED] said:
 Boa noite, pessoal!


 Estava tentando me lembrar das demonstrações das seguintes identidades:
 [...]
 mdc(a,b) = mdc(a+b,mmc(a,b))

 Alguém por acaso se lembra ou sabe como demonstrá-las?
 [...]

Seja d = (a, b). Então a = du, b = dv, (u, v) = 1.

Por outro lado, (a+b, [a,b]) = (du+dv, [du,dv]) = 
(d(u+v), duv) = d(u+v, uv).

Seja p tal que p divide uv. Sem perda de generalidade, p divide u, logo p não 
divide v. Em particular, p não divide u+v, logo p não divide (u+v, uv), logo 
(u+v, uv) = 1.

[]s,

- -- 
Fábio ctg \pi Dias Moreira
-BEGIN PGP SIGNATURE-
Version: GnuPG v1.2.3 (GNU/Linux)

iD8DBQFASaC2alOQFrvzGQoRAgWeAJ0fRZggVmxn4yukRYJt7TtBOBQ3fwCgz4kN
nlnjExcVIrzAMkOFWfFQdyE=
=+XM2
-END PGP SIGNATURE-


=
Instruções para entrar na lista, sair da lista e usar a lista em
http://www.mat.puc-rio.br/~nicolau/olimp/obm-l.html
=


Re: [obm-l] questao de trigonometria - ITA

2004-03-06 Por tôpico Fábio Dias Moreira
-BEGIN PGP SIGNED MESSAGE-
Hash: SHA1

Emanuel Valente [EMAIL PROTECTED] said:
 vacilo cara... nao tinha visto!

 4*sen^2(x) -2*(1+raiz2)*sen(x) + raiz2  0
 [...]

Complete os quadrados: seja t = sen x.

4t^2 - 2*(1+sqrt(2))*t + sqrt(2)  0
(2t - (1+sqrt(2))/2)^2 + (2*sqrt(2)-3)/4  0
[2t - (1+sqrt(2))/2]^2  [(sqrt(2) - 1)/2]^2
|2t - (1+sqrt(2))/2|  (sqrt(2) - 1)/2
|4t - 1 - sqrt(2)|  sqrt(2) - 1
1 - sqrt(2)  4t - 1 - sqrt(2) *e* 4t - 1 - sqrt(2)  sqrt(2) - 1
t  1/2 *e* t  sqrt(2)/2
1/2  t  1/sqrt(2)
1/2  sen x  1/sqrt(2).
pi/6  x  pi/4 *ou* 3*pi/4  x  5*pi/6.

[]s,

- -- 
Fábio ctg \pi Dias Moreira
-BEGIN PGP SIGNATURE-
Version: GnuPG v1.2.3 (GNU/Linux)

iD4DBQFASixralOQFrvzGQoRAtuaAJ4/LKcB1SfMu7DUutRwoNNodLZ5CwCY5010
4i2/ylF4nbxyiTRSiCW44g==
=HTFj
-END PGP SIGNATURE-


=
Instruções para entrar na lista, sair da lista e usar a lista em
http://www.mat.puc-rio.br/~nicolau/olimp/obm-l.html
=


Re: [obm-l] TRIGONOMETRIA

2004-03-05 Por tôpico Fábio Dias Moreira
-BEGIN PGP SIGNED MESSAGE-
Hash: SHA1

Jorge Paulino [EMAIL PROTECTED] said:
 Questão: qual o valor máximo da função y=3cosx+2senx?
 O gabarito é sqrt(13) e eu resolvi usando derivada.
 É possível resolvê-la sem derivada, usando apenas
 conhecimentos do ensino médio?
 Obrigado,
 Jorge
 [...]

Seja S = 3*cos x + 2*sen x. Divida a equação por sqrt(13):

S/sqrt(13) = 3/sqrt(13)*cos x + 2/sqrt(13)*sen x. Seja t tal que sen t = 
3/sqrt(13), cos t = 2/sqrt(13) (como sen^2 t + cos^2 t = 1, certamente existe 
esse t). Logo
S/sqrt(13) = sen t cos x + cos t sen x = sen t+x.
S = sqrt(13) sen t+x

cujo máximo é obviamente sqrt(13).

[]s,

- -- 
Fábio ctg \pi Dias Moreira
-BEGIN PGP SIGNATURE-
Version: GnuPG v1.2.3 (GNU/Linux)

iD8DBQFASS7HalOQFrvzGQoRAvtqAJ90A+q1l7DonFDOY6S0Sc2vbMyfFACfSHY+
mPpb1QOFI+aAIji1BgFxuD4=
=FB8U
-END PGP SIGNATURE-


=
Instruções para entrar na lista, sair da lista e usar a lista em
http://www.mat.puc-rio.br/~nicolau/olimp/obm-l.html
=


Re: [obm-l] Serie...

2004-03-04 Por tôpico Fábio Dias Moreira
-BEGIN PGP SIGNED MESSAGE-
Hash: SHA1

David [EMAIL PROTECTED] said:
 Nao sei ele, mas eu queria assim:

 T(1) = 1
 T(2) = 2
 T(3) = 1
 T(4) = 3
 T(5) = 1
 T(6) = 2
 T(7) = 1
 ...

 Será q eh possivel?
 [...]

T(n) = k+1, onde k é o expoente do 2 na fatoração de n em números primos.

[]s,

- -- 
Fábio ctg \pi Dias Moreira
-BEGIN PGP SIGNATURE-
Version: GnuPG v1.2.3 (GNU/Linux)

iD8DBQFAR0RnalOQFrvzGQoRAvorAKCY4rwoAtDLPZB3Z/7f+3J9d7y2YQCgk5r7
XPk+nzhhY8Fo8NNsNYx/veU=
=bNAh
-END PGP SIGNATURE-


=
Instruções para entrar na lista, sair da lista e usar a lista em
http://www.mat.puc-rio.br/~nicolau/olimp/obm-l.html
=


Re: [obm-l] A^2005 = I == A = I

2004-03-02 Por tôpico Fábio Dias Moreira
-BEGIN PGP SIGNED MESSAGE-
Hash: SHA1

Claudio Buffara [EMAIL PROTECTED] said:
 on 02.03.04 11:36, Nicolau C. Saldanha at [EMAIL PROTECTED] wrote:
  [...]
  Talvez uma versão corrigida do problema do Claudio seja:
 
  Seja A uma matriz 3x3 com coeficientes *racionais* tal que A^2005 = I.
  Prove que A = I.
 
  []s, N.

 Acho que agora deu certo!
 [...]
 Uma duvida: existe uma maneira mais curta de se provar que A = I, dado que
 A^2005 = I e que os autovalores de A sao 1, 1 e 1?
 [...]

Se os autovalores de A são 1, 1 e 1, então A pode ser escrita como P^-1*B*P, 
onde B é uma matriz diagonal cujos elementos são os autovalores de A, i.e. 1, 
1 e 1. Logo B = I == A = P^-1*P == A = I.

[]s,

- -- 
Fábio ctg \pi Dias Moreira
-BEGIN PGP SIGNATURE-
Version: GnuPG v1.2.3 (GNU/Linux)

iD8DBQFARM+ValOQFrvzGQoRAs2VAKCuh9PbQDnpJkFAUYfslWgFQSvufgCglr+r
9EV+LYmA9biVad5DWjtBBIg=
=PrNl
-END PGP SIGNATURE-


=
Instruções para entrar na lista, sair da lista e usar a lista em
http://www.mat.puc-rio.br/~nicolau/olimp/obm-l.html
=


Re: [obm-l] ANALITICA

2004-03-02 Por tôpico Fábio Dias Moreira
-BEGIN PGP SIGNED MESSAGE-
Hash: SHA1

[EMAIL PROTECTED] said:
 Caros amigos, eis uma questão interessante que não sei fazer. Creio que
 seja de analitica, ou talvez, de pra se matar por plana.  Tentem fazer aí
 por obséquio.


 As distâncias dos três vértices de um triângulo a uma reta mede 7 m, 9m e
 14 m. Pode-se afirmar que a distância,em metros, do baricentro do triângulo
 a mesma reta é:

 a) 2/3 b)16/3 c 10 d) pode ser 4
 [...]

Sem perda de generalidade, essa reta é o eixo x. Novamente s.p.d.g., dois dos 
vértices do triângulo estão no semiplano superior (caso isso não seja 
verdade, faça uma reflexão em torno do eixo x).

Há quatro casos:

I) Os três vértices estão no semiplano superior
II) O vértice que dista 7m está no semiplano inferior
III) O vértice que dista 9m está no semiplano inferior
IV) O vértice que dista 14m está no semiplano inferior

Não é muito difícil ver que, medindo as distâncias em metros, os módulos das 
coordenadas y dos vértices do triângulo valem 7, 9 e 14 -- mas em cada um dos 
casos enumerados, os sinais das coordenadas y -- no primeiro, as coordenadas 
y são 7, 9, 14; no segundo, -7, 9, 14; em III, 7, -9, 14 e no último, 7, 9, 
- -14. As coordenadas y do baricentro dos triângulos em cada caso são 10, 16/3, 
2/3 e 4.

Como não temos nenhuma informação sobre qual dos quatro casos é verdadeiro, só 
podemos afirmar que a distância do baricentro ao eixo x (ou seja, o módulo da 
coordenada y) pode valer 10, 16/3, 2/3 ou 4. Logo, como pode valer 4, a 
resposta é a letra (d).

[]s,

- -- 
Fábio ctg \pi Dias Moreira
-BEGIN PGP SIGNATURE-
Version: GnuPG v1.2.3 (GNU/Linux)

iD8DBQFARUjJalOQFrvzGQoRAtmfAJ9pfNgo+UmPrC/YXj81/xDNKMJQzgCg1j81
ntTBpxsmkWIRD2o+PYbDjCo=
=x7Rt
-END PGP SIGNATURE-


=
Instruções para entrar na lista, sair da lista e usar a lista em
http://www.mat.puc-rio.br/~nicolau/olimp/obm-l.html
=


Re: [obm-l] Criterio de divisibilidade

2004-03-01 Por tôpico Fábio Dias Moreira
-BEGIN PGP SIGNED MESSAGE-
Hash: SHA1

Claudio Buffara [EMAIL PROTECTED] said:
 Caros Fabio e Qwert:

 Voces poderiam, por favor, me explicar que criterio estah sendo discutido
 abaixo?
 [...]

Esse aqui:

http://www.mat.puc-rio.br/~nicolau/olimp/obm-l.200306/msg00796.html

[]s,

- -- 
Fábio ctg \pi Dias Moreira
-BEGIN PGP SIGNATURE-
Version: GnuPG v1.2.3 (GNU/Linux)

iD8DBQFAQ7XialOQFrvzGQoRAlFnAKDgTOR1PGORGjF5L/TpU/bHXhSgngCeNW2b
FAEY7lHCi8cCjI3P6vUeQNA=
=zY4W
-END PGP SIGNATURE-


=
Instruções para entrar na lista, sair da lista e usar a lista em
http://www.mat.puc-rio.br/~nicolau/olimp/obm-l.html
=


Re: [obm-l] A^2005 = I == A = I

2004-03-01 Por tôpico Fábio Dias Moreira
-BEGIN PGP SIGNED MESSAGE-
Hash: SHA1

Claudio Buffara [EMAIL PROTECTED] said:
 Oi, pessoal:

 Alguem tem uma solucao elegante pro problema a seguir?

 Seja A uma matriz real 3x3 tal que A^2005 = I.
 Prove que A = I.

 Eu consegui fazer mas achei minha solucao horrorosa. Acho que pode haver
 algum teorema macetoso de algebra linear que eu desconheco.
 [...]

Eu acho que A = [1 0 0; 0 cos(t) sen(t); 0 -sen(t) cos(t)], onde t = 
2*pi/2005, é tal que A^2005 = I -- pelo menos o PARI-GP concorda comigo.

[]s,

- -- 
Fábio ctg \pi Dias Moreira
-BEGIN PGP SIGNATURE-
Version: GnuPG v1.2.3 (GNU/Linux)

iD8DBQFARAWdalOQFrvzGQoRAlOJAJ0dMwlHT7ThUHFlpw58ExWY7YyywACgh4M1
vGd3ZGsvxOBQ4O6c9nGQ388=
=n3AB
-END PGP SIGNATURE-


=
Instruções para entrar na lista, sair da lista e usar a lista em
http://www.mat.puc-rio.br/~nicolau/olimp/obm-l.html
=


Re: [obm-l] Interessante

2004-02-28 Por tôpico Fábio Dias Moreira
-BEGIN PGP SIGNED MESSAGE-
Hash: SHA1

Jefferson Franca [EMAIL PROTECTED] said:
 Ontem um amigo mostrou que uma potência de base 24 e expoente ímpar e
 natural sempre dá como resultado um número que termina em 24! Pq?
 [...]

(mod 100), 24^(2*n+1) = 24 * (24^2)^n = 24 * 76^n. Como 76^2 = 76 (mod 100), 
não é difícil ver que 76^n = 76. Logo 24^(2*n+1) = 24 * 76 = 24.

[]s,

- -- 
Fábio ctg \pi Dias Moreira
-BEGIN PGP SIGNATURE-
Version: GnuPG v1.2.3 (GNU/Linux)

iD8DBQFAQO6+alOQFrvzGQoRAkGHAKCOQNXNPu7Ns0V91jhJUMRdqnR9BACeL/1U
t+9qwJTmHuYiZLn2ucEuJc8=
=GnNp
-END PGP SIGNATURE-


=
Instruções para entrar na lista, sair da lista e usar a lista em
http://www.mat.puc-rio.br/~nicolau/olimp/obm-l.html
=


Re: [obm-l] duvidazinha

2004-02-28 Por tôpico Fábio Dias Moreira
-BEGIN PGP SIGNED MESSAGE-
Hash: SHA1

Tarcio Santiago [EMAIL PROTECTED] said:
 QUAL A CONDIÇÃO PAR QUE O ANO SEJA BISSEXTO?

Se um ano for divisível por 4, mas não por 100, ele é bissexto.
Se um ano for divisível por 400, ele é bissexto.
Todos os outros anos não são bissextos.

Há propostas que fazem anos divisíveis por 4000 não bissextos, mas, a menos 
que você esteja falando de eventos geológicos ous astronômicos, isso não faz 
a menor diferença.

Referência: http://en.wikipedia.org/wiki/Leap_year

[]s,

- -- 
Fábio ctg \pi Dias Moreira
-BEGIN PGP SIGNATURE-
Version: GnuPG v1.2.3 (GNU/Linux)

iD8DBQFAQXxbalOQFrvzGQoRAldxAKCPzdCIebo0sI+RbMQmnHFwLSy78wCfUSRs
Lvi4ros609iVRAwS7x9GCNg=
=g9Da
-END PGP SIGNATURE-


=
Instruções para entrar na lista, sair da lista e usar a lista em
http://www.mat.puc-rio.br/~nicolau/olimp/obm-l.html
=


Re: [obm-l] Número Primo + duvida

2004-02-27 Por tôpico Fábio Dias Moreira
-BEGIN PGP SIGNED MESSAGE-
Hash: SHA1

Qwert Smith [EMAIL PROTECTED] said:
 Fiquei com uma duvida... vou tentar explicar:

 No caso particular do numero 7919 tem uma maneira bem facil (dependendo da
 resposta a minha duvida) ja que 7919 termina em 9 e nessa lista mesmo ja
 foi mostrado como determinar o criterio de divisibilidade de primos
 terminados em 9.

 7919 = 791*10 + 9 entao temos:
 se 7919 e primo entao para verificar se (10a + b) e multiplo de 7919 basta
 verificar se (a + 792*b) e multiplo de 7919.  Parece complicado, mas o
 teste e bem simples.
 [...]
 aki fica a duvida...
 Sei que se a regra de divisibilidade NAO pode ser aplicada com sucesso para
 um numero que termina em 9, esse numero NAO e primo, mas dar certo eh o
 suficiente pra afirmar que o numero com certeza e primo?
 [...]

Não -- a regra funciona para qualquer número primo com 10, composto ou não.

[]s,

- -- 
Fábio ctg \pi Dias Moreira
-BEGIN PGP SIGNATURE-
Version: GnuPG v1.2.3 (GNU/Linux)

iD8DBQFAP3ZOalOQFrvzGQoRAnmFAKCuyh0jSS9HyYZOwkrGEc6ayMvbugCdG76X
hgUvkQEnI0TZqQxI4jZSo0Y=
=vM3y
-END PGP SIGNATURE-


=
Instruções para entrar na lista, sair da lista e usar a lista em
http://www.mat.puc-rio.br/~nicolau/olimp/obm-l.html
=


Re: [obm-l] Existe uma solução mais simples?

2004-02-27 Por tôpico Fábio Dias Moreira
-BEGIN PGP SIGNED MESSAGE-
Hash: SHA1

Danilo notes [EMAIL PROTECTED] said:
 Pessoal  segue abaixo um problema que achei muito interessante e a forma
 como fiz pra resolve-lo, gostaria de saber se alguem conhece uma solução
 mais simples. Seja A uma matriz quadrada  n x n tal que  3A^3=A^2+ A + I 
 prove que  ( A^k)  converge para  B  tal que   B^2=B .   k   é numero  
 natural. Solução:
 Seja  p(x) =3x^3-x^2-x-1   .   Vamos considerar 2 casos:
 i)A é uma matriz real .

 Observe que  P(x) =( x-1) ( 3x^2+2x+1), como A é matriz real  e  P(A)=0 
 então  A =I = matriz identidade e daí é imediato que   lim ( A^k) =I =B   e
   B^2=B
 [...]

O conjunto das matrizes nxn não forma um domínio. Ou seja, AB = 0 *não* 
implica A = 0 ou B = 0.

Por isso (A-I)(3A^2 + 2A + 1) = 0 *não* implica A-I = 0 ou 3A^2 + 2A + 1 = 0. 
Logo não podemos afirmar que A = I.

[]s,

- -- 
Fábio ctg \pi Dias Moreira
-BEGIN PGP SIGNATURE-
Version: GnuPG v1.2.3 (GNU/Linux)

iD8DBQFAP62kalOQFrvzGQoRAuSFAKCjaaVT4vnEvZZl/InyAd94sgEdbQCgs6Pk
Vxt9di/qgpruF/yL5VRef90=
=uMIj
-END PGP SIGNATURE-


=
Instruções para entrar na lista, sair da lista e usar a lista em
http://www.mat.puc-rio.br/~nicolau/olimp/obm-l.html
=


Re: [obm-l] Probabilidade

2004-02-27 Por tôpico Fábio Dias Moreira
-BEGIN PGP SIGNED MESSAGE-
Hash: SHA1

Pacini  bores [EMAIL PROTECTED] said:
 Poderiam me  ajudar ?

 Em um grupo de 60 pessoas  qual  a probabilidade  de haver  duas  pessoas
 que  nasceram  no  mesmo  dia  do ano ?
 [...]

Supondo que não existem anos bissextos e que a distribuição dos nascimentos 
durante um mesmo ano é uniforme, vamos calcular a probabilidade de que n 
pessoas *não* compartilhem aniversários.

Cada pessoa possui 365 possíveis dias para aniversariar, logo o número de 
combinações possíveis é 365^n. Por outro lado, o número de combinações que 
não possuem duas pessoas fazendo aniversário no mesmo dia é C(n;365)*n!, pois 
temos que escolher os dias em que fazem aniversário, e a ordem em que elas o 
fazem. Logo, se p_n é a probabilidade de que duas pessoas façam aniversário 
no mesmo dia,

1 - p_n = [365!*n!]/[(365-n)!*n!*365^n] = 365!/[(365-n)!*365^n]

p_n = 1 - 365!/[(365-n)!*365^n]

Para n = 60, isso dá aproximadamente 99,4%.

Naturalmente, você não vai calcular 365! para achar esse valor. Ao invés 
disso, a gente pode reescrever p_n como

1 - 365*364*...*(366-n)/365*365*...*365 = 
1 - (365/365)*(364/365)*...*[(366-n)/365], que é *muito* mais rápido de se 
calcular.

[]s,

- -- 
Fábio ctg \pi Dias Moreira
-BEGIN PGP SIGNATURE-
Version: GnuPG v1.2.3 (GNU/Linux)

iD8DBQFAP8kwalOQFrvzGQoRAglgAJ9q5Al4lX2IQt4Avcc3tLORsEOjDQCgi5aM
oWC0kyh9aceKmLfcml+b1oM=
=fuuA
-END PGP SIGNATURE-


=
Instruções para entrar na lista, sair da lista e usar a lista em
http://www.mat.puc-rio.br/~nicolau/olimp/obm-l.html
=


Re: [obm-l] Relaçoes

2004-02-27 Por tôpico Fábio Dias Moreira
-BEGIN PGP SIGNED MESSAGE-
Hash: SHA1

Thor [EMAIL PROTECTED] said:
 Como mostrar que : dados dois conjuntos A e B não vazios,

 n( R) = 2^n(A).2^n(B), ou seja o nº de relaçoes de A e B eh

 dois elevado ao nº de elementos de A  vezes dois elevado ao nº de elementos
 de B.
 [...]

Essa fórmula me parece estar errada: As únicas relações de {1} em {1} são {} e 
{(1, 1)}, logo 2 = 2^1*2^1 = 4.

A fórmula correta deve ser |R| = 2^(|A|*|B|). Como toda relação é um 
subconjunto de A*B, e vice-versa, R = P(A*B). Logo

|R| = |P(A*B)| = 2^(|A*B|) = 2^(|A|*|B|).

[]s,

- -- 
Fábio ctg \pi Dias Moreira
-BEGIN PGP SIGNATURE-
Version: GnuPG v1.2.3 (GNU/Linux)

iD8DBQFAP+9palOQFrvzGQoRAlumAKDSdWC6yBL/HEiTeJuI0hD4bljIsgCgz3l2
APU49U3Rde8IWRg7AJm/B8s=
=6Vlg
-END PGP SIGNATURE-


=
Instruções para entrar na lista, sair da lista e usar a lista em
http://www.mat.puc-rio.br/~nicolau/olimp/obm-l.html
=


Re: [obm-l] Re: [obm-l] Re: [obm-l] COMISSÃO DE FRENTE!

2004-02-25 Por tôpico Fábio Dias Moreira
-BEGIN PGP SIGNED MESSAGE-
Hash: SHA1

ponciomineiro [EMAIL PROTECTED] said:
 [...]
 On Mon, Feb 23, 2004 at 10:54:54PM -0300, [EMAIL PROTECTED] wrote:
 Três pontos são selecionados aleatóriamente numa circunferência de raio
 unitário. Encontre a probabilidade de esses pontos pertencerem a uma mesma
 semicircunferência.
 [...]

 Nicolau e amigos da lista, boa noite!!!
 Gostaria de solicitar comentários acerca do modo que acredito ter
 resolvido a questão:
 Considere A e B dois pontos não diametralmente opostos tomados nessa
 circunferência (caso fossem diametralmente opostos seria evidente que os
 três pontos pertenceriam a uma mesma semicircunferência). Considere, ainda,
 A* e B* pontos diametralmente opostos a A e B, respectivamente. Observe
 que, dessa forma, divido a circunferência nos arcos AB, BA*, A*B* e B*A.
 Portanto, o terceiro ponto estaria em uma mesma semicircunferência dos
 outros dois caso não pertencesse a A*B* (excluindo A* e B*, evidentemente).
 Logo, a probabilidade pedida seria 3/4.
 [...]

Isso estaria certo se os quatro arcos sempre tivessem o mesmo comprimento. 
Como não têm, as probabilidades associadas a cada um deles não é a mesma, 
logo a probabilidade pedida não é 3/4.

[]s,

- -- 
Fábio ctg \pi Dias Moreira
-BEGIN PGP SIGNATURE-
Version: GnuPG v1.2.3 (GNU/Linux)

iD8DBQFAPUPRalOQFrvzGQoRApHxAKDcGXFkP+7tN/CxGAxiOPxy9GTsvACfWVuD
5wC9pwabmU0D+2VDIIEYHW0=
=3kUi
-END PGP SIGNATURE-


=
Instruções para entrar na lista, sair da lista e usar a lista em
http://www.mat.puc-rio.br/~nicolau/olimp/obm-l.html
=


Re: [obm-l] dúvidas

2004-02-24 Por tôpico Fábio Dias Moreira
-BEGIN PGP SIGNED MESSAGE-
Hash: SHA1

[Wednesday 25 February 2004 01:59: [EMAIL PROTECTED]]
 olá amigos poderiAM DAR UMA AJUDA POR FAVOR
 [...]
  2- Dois mísseis são lançados diretamente um contra o outro, oprimeiro a
 18.000 km/hora e o segundo a 12.000 km/hora. Sabendoque no instante do
 lançamento eles se encontravam a 4768quilômetros de distância um do outro,
 a distância entre eles, a umminuto da colisão é, em kilômetros:

 (A) 500(B) 750(C) 1000(D) 1500(E) 2384
 [...]

Como, de um minuto antes da colisão até a colisão, passou-se um minuto, o 
primeiro míssil percorreu 18000/60 = 300 Km e o segundo 12000/60 = 200 Km. 
Logo a distância entre os dois mísseis um minuto antes de colidirem é de 500 
Km.

[]s,

- -- 
Fábio ctg \pi Dias Moreira
-BEGIN PGP SIGNATURE-
Version: GnuPG v1.2.3 (GNU/Linux)

iD8DBQFAPDLzalOQFrvzGQoRAsNmAKDGrCY9/NpBJStkrdXudCM0PEKq8QCfcJ/Z
an44rIqU/7TFRwaw0HQQkP4=
=Ulv7
-END PGP SIGNATURE-


=
Instruções para entrar na lista, sair da lista e usar a lista em
http://www.mat.puc-rio.br/~nicolau/olimp/obm-l.html
=


Re: [obm-l] Correção -Questões

2004-02-22 Por tôpico Fábio Dias Moreira
-BEGIN PGP SIGNED MESSAGE-
Hash: SHA1

[Saturday 21 February 2004 21:56: [EMAIL PROTECTED]
 Descupem , o certo  para o problema  2 é :

 2)Quais  os  dois  últimos  algarismos   na  parte inteira de

 10^2047/(10^89 +7).
 [...]

Note que 10^2047 = (10^89)^23. Por isso, 10^2047 = (10^89)^23 (mod 10^89 + 7). 
Portanto, 10^2047 = (-7)^23 = -7^23 = 10^89 + 7 - 7^23 (mod 10^89 + 7). Como 
este último número é obviamente positivo e menor do que 10^89 + 7, ele é o 
resto da divisão de 10^2047 por 10^89 + 7. Por isso, a parte inteira de 
10^2047/(10^89 + 7) é igual a

N = (10^2047 + 7^23 - 7 - 10^89)/(10^89 + 7). Como queremos os dois últimos 
algarismos, basta fazer a conta módulo 100. Mas N (mod 100) vale
(7^23 - 7)/7 = 7^22 - 1. Como 7^4 = 1 (mod 100), N = 7^22 - 1 = (7^4)^5*7^2 - 
1 = 1^5*7^2 - 1 = 7^2 - 1 = 48.

[]s,

- -- 
Fábio ctg \pi Dias Moreira
-BEGIN PGP SIGNATURE-
Version: GnuPG v1.2.3 (GNU/Linux)

iD8DBQFAONoValOQFrvzGQoRAmoHAJ94piJD7r44xvaFR/B6Pwn0y+t7BgCfUDRp
DPd58SXO6Ewf1rusPGpLOJg=
=EZGe
-END PGP SIGNATURE-


=
Instruções para entrar na lista, sair da lista e usar a lista em
http://www.mat.puc-rio.br/~nicolau/olimp/obm-l.html
=


Re: [obm-l] restos

2004-02-22 Por tôpico Fábio Dias Moreira
-BEGIN PGP SIGNED MESSAGE-
Hash: SHA1

[Sunday 22 February 2004 23:41: OBM-L [EMAIL PROTECTED]]
 Rafael,

 Para o primeiro problema, seja D o dividendo e N o maior inteiro que possa
 ser somado a D para que o quociente Q não sofra alteração, temos:

 N / 13 = Q (mod 2) == (N+10) / 13 = Q (mod 12), pois se D = 11, teríamos
 (Q+1) como quociente. Logo, D = 10.

 Já quanto ao segundo problema, não entendi uma coisa: como um número
 dividido por 3 pode deixar resto 3? Se isso estiver certo, significa que a
 divisão é exata para 3 e deixa resto 3 para 11 e 51 (que é 17*3). Se a
 divisão é exata para 3, então o menor número é um múltiplo de 3 que deixa
 resto 3 para 11 e 51. Logo, o mmc(3;11;17) = 561 e 561+3 = 564.
 [...]

Posso estar enganado, mas eu acho que o menor múltiplo de 3 que deixa resto 3 
quando dividido por 11 e 51 é 3.

[]s,

- -- 
Fábio ctg \pi Dias Moreira
-BEGIN PGP SIGNATURE-
Version: GnuPG v1.2.3 (GNU/Linux)

iD8DBQFAOXFGalOQFrvzGQoRAsZDAJ9vYmeH47jGUia8AzIQyztP2gCZnwCggPwz
yxhrG+YI1QGqZNRBkyeKaz0=
=coOF
-END PGP SIGNATURE-


=
Instruções para entrar na lista, sair da lista e usar a lista em
http://www.mat.puc-rio.br/~nicolau/olimp/obm-l.html
=


Re: [obm-l] questões

2004-02-21 Por tôpico Fábio Dias Moreira
-BEGIN PGP SIGNED MESSAGE-
Hash: SHA1

[Saturday 21 February 2004 21:00: [EMAIL PROTECTED]
 Estou  emperrado  nos  seguintes problemas :

 1)Qual o resto  de 2269^n+1730^n + 1779^n-1776^n  na divisão por 2001 ?
 sendo   n ímpar . É claro  que  para n=1  temos 4002  para  a soma ,
 mas  para  um n ímpar  qualquer  ?
 [...]

Note que 2001 = 3*23*29, logo, se f(n) = 2269^n + 1730^n + 1779^n - 1776^n, 
basta achar f(n) mod 3, 23 e 29 -- podemos combinar os resultados depois com 
o Teorema Chinês dos Restos.

Módulo 3, f(n) é 1^n + (-1)^n + 0^n - 0^n = 1 + -1 + 0 + 0 = 0.

Módulo 23, f(n) é (-8)^n + 5^n + 8^n - 5^n. Como n é ímpar, (-8)^n = -8^n, 
logo f(n) = 0 (mod 23).

Módulo 29, f(n) é 7^n + (-10)^n + 10^n - 7^n. Novamente, como n é ímpar, f(n) 
= 0 (mod 29).

Logo 2001 sempre divide f(n), i.e. o resto da divisão de f(n) por 2001 é zero.

[]s,

- -- 
Fábio ctg \pi Dias Moreira
-BEGIN PGP SIGNATURE-
Version: GnuPG v1.2.3 (GNU/Linux)

iD8DBQFAN/g6alOQFrvzGQoRAuRvAJ9HHwlpzyeY+FYkExBSI6JCfRGELACg0JKG
KZfm0N2qL2/A+s7+bG0UOS0=
=CL5f
-END PGP SIGNATURE-


=
Instruções para entrar na lista, sair da lista e usar a lista em
http://www.mat.puc-rio.br/~nicolau/olimp/obm-l.html
=


Re: [obm-l] Resolução da função...

2004-02-18 Por tôpico Fábio Dias Moreira
-BEGIN PGP SIGNED MESSAGE-
Hash: SHA1

[Wednesday 18 February 2004 10:52: [EMAIL PROTECTED]
 f(x) = x^2 + (1 - sqrt 3)x - sqrt 3

 Delta = (1 - sqrt 3)^2 - 4(1)(- sqrt 3)
 = 1 - 2(sqrt 3) + 3 + 4(sqrt 3)
   = 4 + 2(sqrt 3)

 x = (- (1 - sqrt 3) + ou - sqrt(4 + 2(sqrt 3))) / 2
= (-1 + sqrt 3 + ou - sqrt(4 + 2(sqrt 3))) / 2

 x1 = (-1 + sqrt 3 + sqrt(4 + 2(sqrt 3))) / 2
 x2 = (-1 + sqrt 3 - sqrt(4 + 2(sqrt 3))) / 2

 onde está o erro?? ou o q está faltando??
 no livro do Iezzi a resposta é (sqrt 2) / 2
 [...]

Note que sqrt(4+2*sqrt(3)) = 1 + sqrt(3), logo x1 = sqrt(3) e x2 = -1.

[]s,

- -- 
Fábio ctg \pi Dias Moreira
-BEGIN PGP SIGNATURE-
Version: GnuPG v1.2.3 (GNU/Linux)

iD8DBQFAM4R1alOQFrvzGQoRAgaAAKDhltkcZZM2o2ZxJF6trzeYmly6jACfZR6z
J6jYZ5nah79Nh41uEwbLQjw=
=udXC
-END PGP SIGNATURE-


=
Instruções para entrar na lista, sair da lista e usar a lista em
http://www.mat.puc-rio.br/~nicolau/olimp/obm-l.html
=


Re: [obm-l] Ajuda

2004-02-15 Por tôpico Fábio Dias Moreira
-BEGIN PGP SIGNED MESSAGE-
Hash: SHA1

[Sunday 15 February 2004 15:35: [EMAIL PROTECTED]]
 Qual das duas coleções de livros para 2º Grau os amigos consideram melhor:

 A coleção do Iezzi, ou a do Manoel Paiva.

 Se houver alguma outra que seja melhor que as duas por favor citem.
 [...]

Uma referência útil é o Exame de Textos (leia os capítulos relevantes):

http://www.ensinomedio.impa.br/materiais/analise_de_text/

[]s,

- -- 
Fábio ctg \pi Dias Moreira
-BEGIN PGP SIGNATURE-
Version: GnuPG v1.2.3 (GNU/Linux)

iD8DBQFAL7iYalOQFrvzGQoRAra8AKC/kqdDmPl0pv85XjiFcox44R8ZZACeOCYE
zpWNKEclIiKxxsAxT9gQz4A=
=nCmW
-END PGP SIGNATURE-


=
Instruções para entrar na lista, sair da lista e usar a lista em
http://www.mat.puc-rio.br/~nicolau/olimp/obm-l.html
=


  1   2   >